Sie sind auf Seite 1von 94

MEDICINE

Pt admitted to hosp due to gentourianry infection not sure though! History of rash to
penicillin. staph aureus on culture which is the antibiotic of choice for her
A.Ciprofloxacin
B.Cefazoline
C.Cefalaxine
D.vancomycin
E.ceftriaxone

.old lady was found lying on the ground by her neighbours….she was drowsy and dehydrated…noticed dark urine
when urinary catheter inserted ,hematoma on thigh ….weight 42kg
Blood investigations given CK was elevated 11000
What is the next step of management?
a) Normal saline with urinary output 2ml/kg/hr
b) Dextrose 1/5 saline with urinary output 1ml/kg/hr
c) 5% dextrose with urinary output 2ml/kg/hr
d) Normal saline with urinary output 1ml/kg/hr
dehydration or rhabdo

A male pt came with cervical injury. BP is 90/50.HR 50.What will u give?


a. I/V colloid solution
b. Trendelenburg position
c. blood transfusion
d. adrenalin

Clinical signs of Trendelenburg?


a. ipsilateral shortening of leg
b. contralateral shortening of leg
c. ipsilateral abductor weakness
d. mechanism of imbalance

– very weired question about pt with low MCV and anaemia 8.6 , had fever for 3 days 2 weeks before presentation
that subsided spontaneously , now presents with anaemia investigation asked , no bilirubin given
Autoimmune haemolytic anaemia(coomb’s test)
Malaria (thick and thin film)

multiple medications use , low mcv , cause :


-asprin
-diclophinac
-chloriquene

Among all which drug is hepatotoxic drug require liver function test. a.) Venaflaxine b.) Hydrocholorothiazide c.)
Furesimide d.) Gentamycin Q

The damage to the trigeminal nerve can cause ? a.) Forhead wrinkling b.) Vision loss c.) Squint or double vision d.)
Loss of taste e.) Difficulty in opening a jaw q.

Pt with renal functions impaired, v/Q scan was done, showed some thing in rt lung, like positive finding for PE
whats next.
a.Warfarin
b.Iv heprin
c.s/c enoxaparin
d.LMWH
Low molecular weight heparin is therefore usually the agent of choice for initial treatment of PE.
Unfractionated heparin is still preferred in patients with significant renal impairment, due to the renal
clearance of LMWH
Doppler showing plaque from tibial to popiliteal vein, patient RFts were very dearranged.
a.Warfarin
b.Iv heprin
c.s/c enoxaparin
d.LMWH jm 1337

pt 4 days after parathioroid sx for parathyroid adenoma, develop finger and perioral numbness ca level 2 days back
at time of discharge was 2.02 (it was low then the normal limits given) was low wt to give
a-ca carbonate
b- ca carbonate and vit d3-since mild
c.calcitrol
calcium iv-if severe

Pregnant women first trimester come to you want to stop smoking but can't, she smokes 25 segrat/ day
what will you Give?
A. Non nicotine chewing gum" non Nicotine"
B. Nicotine therapeutic therapy
First try therapy and support then nicotine replacement can be given aft
explaining benefits and disadvantages …best nrt is lozenges or sprays

old Pt come with 10 months history of reflux at night, epigastric tenderness, loves spicy
food,smoking 60 cigarettes,drinks 15 standard drink/ day of alcohol, he is obese waist
circumference was high and asking what will u advice for long term mx
A. Stop alcohol
B. Stop smoking
C. Avoid spicy food
D. Reduce obesity
E. Long term ppi

Pt came with fever, weakness, fatigue, some infection, lab Ix-Hb reduced, WBC increased,
Neutrophil increased, Dx asked-
ALL
CML
Multiple myeloma

old pt with hamaturia ,increase urea ,creatine ,arthralgia,foot drop,what will lead u to dx:
a. Sural nerve biopsy-pan
b. Renal biopsy
c. ANCA

Microscopic polyangiitis (Dx Given), asking for Ix-


Anti CCP
ANA
ANCA

. Patient came with penicillin allergy, what’s most important thing to consider ?
a. Time of onset
b. Fever
c. Itchiness
d. Red papule

flial chest Trauma to chest by MVA and flail chest and open chest wound ,important step to be
done at scene(accident site)
A-Morphine IV
B_pressure by gauze to close the wound
C_chest tube
D_chest strapping

Patient is taking methotrexate, prednisolone and celecoxib for RA. Lab results show
pancytopenia with high esr (67). Management?
Increased prednisolone
HCQ
Folic acid – along with mtx
Folinic acid-always give e mxt
Cease celecoxib

A 49 year old female presented with progressive back pain. She has a history of Breast Ca and
lumbar osteoarthritis. What of the following clinical features will help you decide the
investigation (MRI) to do?
Past history of breast Ca
Back pain without trauma
Past history of vertebral osteoarthritis
High blood pressure
Radicular Pain radiating to the buttock

Man presents with one alopecia patch on scalp and rash in hands and legs. He recently travelled
asia alone and also live in Darwin for 3 months. What is initial investigation for his condition?
Syphilis serology
Thyroid function test
Serum zinc level
Malaria blood film
Don’t remember now and it is blood test too

Sarcoidosis xray with nodules - dyspnea, 12months loin pain, 24hours hematuria…what inv
have to do NEXT?
a) CT chest b)Serum ACE c)USG of abdomen d) urine r/m/e e)cystoscopy -

A 68 years old man with COPD was brought to your surgery from nursing home by
ambulance. On his way to hospital he received O2, 10 L/m by mask. He is still
unarousable and his ABG most likely-- a) Ph 7.29 PaCO2 65 PaO2 85 resp acidosis b) Ph
7.15 PaCO2 50 PaO2 68 c) Ph 7.25 PaCO2 25 PaO2 100

A 16-year old female comes in for treatment of severe cystic acne with Accutane(isotretinoin).
She lives alone and is self-supporting with a job as a waitress. She hasbeen out of her
parents'house for a year and pays all her own bills. you have just fin_ishecl informi'g her of the
potentially sevcre teratogenicitv of isotretinoin. Her acneis severe and she would still like the
isotrctinoin.What should you tell her?
a.This medication cant be taken by women of reproductive age
b. Use benzol4 peroxidc topically instead
c. l will give you the isotretir.roin if your parents accompany you
d. I will give you the isotretinoin as requested.
e. I will treat you with isotretinoin ifyou have a consent signed by your parents

. Which one of the following can be used for data analysis in Cross sectional?
Odds ratio
Relative risk
Chi square test
Attributable risk and Chi square test
odds ratio and attributable risk

Which of the follorving most closely represents the role of risk managernent in the hospital
a to ensure proper ethical management ofpatients
b. To ensure proper clinical care ofpatients
c. To act as a patient advocated.
D. To minimize the legal risk to the hospital from litigation

There is a photo of leg ulcer medial side of the leg with dark discoloration blue color extending
to a large area of the leg, I saw some yellow crusts surrounding the ulcer, the ulcer itself was
above the medial malleolus, pt has Hx of DVT, DM long standing, peripheral pulses absent,
asking about the cause of discoloration
a. diabetic ulcer
b. venous ulcer
c. arterial ulcer
d. hemosiderin deposition
e. bleeding into the muscle layers
There is Q pa has repetitive previous Hx of coliky abdominal pain description fits with renal
stones, he came with SOB and cough. CXR given lots of haziness bilateral no mass , no
cardiomegaly, investigation asked to reach to a Dx.
a. Ca level
b. CT chest
c. CT abdomen
d. echo
e. forget….
35ys old man brought by wife, presented with confusion, ataxia, right upper abdominal pain,
changed behavior, memory loss, weakness, pt drink alcohol and smoke some per day, T 37.5,
vitals normal.
He has his father and paternal aunt has same history when at his age. Asked what investigation
to reach Dx
a. CT brain
b. CT abdomen
c. MRI brain
d. Check serum ceruloplasmin-wilsons

What is the U/S benefit used as abdominal diagnostic measure


a. injury to solid organs
b. to Dx fluid filled cyst
c. To Dx ruptured viscus

Gbs stem.how to monitor respiration.


FEV
FVC
Monitor chest expansion

Post op incarcerated hernia 1st day aggitation stem fever sob O2 sat 88% xray given after o2
what initial step.xray had patches!(atelectasis) What initial
IV antibiotics
Heparin
Thrombolysis
Droperidol

Young boy developed pain in the right knee .. mild progressive marked swelling .. no other
systemic symptoms as I recall Blood results given .. anemia .. low platelets .. normal WBCs What
is your diagnosis
Juvenile rheumatoid arthritis -multiple joints and
Acute leukemia
Apalstic anemia-pancytopenia
SLE-no other feats present
Post op patient after hemicolectomy presents with a yellowish brown discharge. Cause asked
Clostridiim welchi
Staph epidermidis
Strep pnuemonia
Staph aureus
Pseudomonas

While on Aspirin daily 100mg, 55 yr old developed right hemiparesis, which completely resolved
within 8hrs. further evaluation revealed a Left carotid stenosis of 55%. which one of the
following is the most important step in mx:
a)add heparin
b) add cloprez
c)carotid endarterectomy jm 1433
d)increase dose aspirin
e) carotid stenting
35-patient with type 2 diabetes on metformin 500 mg came to routine health checkup, lab
investigation was done showed as follows Test result Random blood sugar 5.6 (4.5-6.5)
Cholesterol (within normal) HbA1c 6.9% ( normal <6.5%) Urine dipstick trace of protein What is
your most appropriate next step in management? A-Commence insulin B-increase metformin C-
Ramipril D-add simvastatin E-continue same treatment

Papillary carcinoma in left lobe, management: Thyroidectomy, Left lobe thyroidectomy

Old age woman on multiple drugs digoxin ACEI BB ,she stopped all drugs 3 weeks ago as she
was in a trip and ran out of drugs ,she is complaining of confusion ,and occasional postural
dizziness (the exact words ) ..lab results show :
Creatinine 118 (normal up to 90 ) Non fasting blood glucose : 4
Bl p. sitting 168/70 ,standing 130 /60 What is the cause :
a.dehydration .
b.renal impairment
c.hypoglycemia

past history of dvt pt on UFH perioopertively n switch to lmw heparin after 5 days undergone rt
hip surgery develop dvt after 10 days that what to do
In investigation only platelets r decreased
A- ffps
B- vit k
C- cease heparin and switch to other anticoagulant heparin induced thrombocytopenia
D- platelets infusion
Yr old man went through hemicolectomy after 2 days he was confused his labs showed low
sodium low potassium low urea low creatinine osmolarity was normal upper limit.diagnosis?
Siadh
Hyperaldosteronism
Hyponatremia
sampling from iv cannula site
women previously investigted for breast cancer, now having complain of bilateral arm
weakness while combing and redness on the skin of the hand, what to do ???
A- EMG- dermatomyositis
b- NCS
C-PET
D- BONE SCAN
E-MRI

you are a dentist and want to relationship between mercury amalgam and dementia. You take
two groups of people one with dementia taking amalgam, one with dementia not taking
amalgam. What is this type of study called?
a) Cross-sectional
b) Randomized controlled trial
c) Case-control
d) Cohort
e) RCT

..Middle yr old man recently arrived from Africa with complaint of Fever , haematuria and
splenomegaly Hb is 8 . He took on dose albendazole and tested malaria that was negative 3
days before he left from Africa .
A) recheck malaria immunoassay
B ) schistosomiasis
C) giardiasis

.Girl comes to cliic C/o just one genital ulcer U did syphilis & other screening she didn't come
after that for 2weeks
now come with for test result
Her viral culture is all negative but RPR test is 1:64 titre positive
how will u Mx?
A.test for other STD
B.contact tracing
C.advice her to use condom
D.HIV testing
E.Notify local public health

Pregnant woman complaint of palpitation, lab result- TSH ↑, next investigation?


Repeat TSH***
Thyroid antibody
Thyroid scan
Thyroid USG
Give thyroxine-if tsh is >10mlIU/L then irrespective of t4 give thyroxine as it is overt
hyputhyroidism
Dr works at residential aged care. Asks for change in patients hypertensive medication from
ACEI to Ca Blocker. Nurse advises Dr that this patient did not respond to Ca Blocker 2 years ago,
and instead recommends use of ACEI and a diuretic. What do you do?
- Take nurses advises ANS
- Report her to Nurses manager
- Incident report
- Report her to some other place.

Man with 3 weeks (or months?) hx of ulcer in his gum, it is red and irritated. The dentures are
poor fitting and loose. What is the cause of ulcer?
- Trauma ANS
- SCC
- Don’t remember

Man presents to ED with wife and another family member. Wife is concerned as he has been
aggressive and has smashed windows today and the other day threatened wife he will kill her.
On exam he said he had few beers at work but his breath smells of strong alcohol. What would
further hint towards his condition?
- Has to have alcohol first thing in the morning before going to work ans
- Has been arrested in the past
- Sexually abused between 6-8 yo
- Some other ones

Pain and erythema with midline tenderness on calf in a man. What is the best management? (Is
the DVT? Midline tenderness!!!)
- Unfractionated heparin ans
- Elevate leg and rest
- Ice
- Analgesia

Old woman weakness in all limbs and hyponatreamic and hypokalemic and I think glucose was
elevated, What is it?
- Siadh not possible con potassium is normal
- Conns not possibe coz hypernatremia
- Some other ones
Diuretic(thiazide and loop) best option if present
Hyperglycemia 2nd best if in options
Long term steroid use
Water intoxication

https://patient.info/health/thiazide-diuretics
A Diabetic female,scratched by cat in upper left arm . Develooed rash n pain in arm , forearm n
hand along with numbness in hand. Diagnosis
Diabetic myotrophy-not in upper limbs
Cat scratch fever-mayb
Cellulitis-no numbness

Lean woman with ten year hx of diabetes type II with high glucose (I think BSL was 16) and
glucosuria, protein, and ketonuria. She takes gliclazide for diabetes. what’s the cause of these
sx?
- Insulin resitance
- Renal failure
- Insufficient insulin production ans
- Two other ones

Patient is marrying her cousin; her friends have told her that there is danger in marrying family
members. She presents to GP asking for information regarding these dangers. What is the best
advise?
- There will be an increased risk of recessive disorders ans
- Do genetic testing once pregnant
- Some other ones
First cousins 12.6%%
2nd cousins 3%%
Double 1st cousin 25%
Double 2nd cousin 6%
Half first cousin 6%

After a night party a girl comes to the ER in the moring with a high fever of 40c, agitated,
decreased consciousness level, muscle rigidity, tremor. What will be the best initial step in
management?
I/V infusion
Cool blanket and ice pack
I/V Diazepam
Dantrolene Sodium

Which group is the less risk to get Hepatitis C (Sure not HIV)
A Drug abuse
B Homosexual,
C Transfusion Hep B,
D Haemodialysis,
E Haemophilia

Study for vit D difficiency in ur community?


A) cross-sectional study
B) cohort study
C) case study
D) case series
E) rct

25 yr boy complain of fatigue,cousine has some blood disorder,require frequent blood


transfusion.on pbf there was mentioned target cell found.what will u do next most app?
s.ferritin
hb electrophoresis-hemoglobinopathies

https://medlineplus.gov/ency/article/003665.htm

african come for refugee health check up.malaria neg two days prior coming here.mentioned
eosinophilia.but no fever mentioned.what most app next inv u will do?
Stool for ova,parasites
Schistosomiasis serology
Check malaria again

girl has hematuria,hemoptysis,progressive distress for 2 months.ix lead you to dx?


s.ace
anti gbm antibody
ct chest

A guy came back from Africa, high fever, you check for malaria, took antimalarics, twice
negative results for malaria, the test has 98% sensitivity, 96% specificity, what to do next?
Check for different disease, blood smear-thick and thin, test in high peak of fever

Pt came with swelling in the leg , duplex was done showed DVT he was started on
enoxaparine(1mg/kg) and warfarine 5 mg after 5 days the pain in his leg increased his INR 1.7 ,
what is the next appropriate step?
Change warfarine to enoximab
Increase warfarine
Change enoxaparine to IV heparin
Repeat US
Correct ans should be continue warfarin
https://vitualis.wordpress.com/2006/06/16/how-to-start-warfarin-therapy/

https://www.health.qld.gov.au/clinical-practice/guidelines-procedures/medicines?a=165945
22.traveller came back from Africa.have fever(not very high)with tender splenomegaly.no gi
symptoms.hb is 8 g/dl (12-15g/gl).malaria test 3 days before he left was negative.(no history of
taking prophylaxix or net using or any h/o bathing in lake…as in previous recalls).what to do
next
a.repeat malarial serology
b.schistosomiasis serology
c.Entamebae histolytica
d.giardiasis

boy 7 yr old with recurrent abscesses.each time staph aureus positive.low weight and height
acc to age.o/e pharyngitis gingivitis and multiple scabs of abscesses.what is helpful in
diagnosing the underlying cause.
a.lymphocyte count
b.neutrophil function
c.complement level
d.antibody level

11.PIC of hand with some red lesions,scenario of a girl with arthralgia and lesions.what is the
long term treartment,
She was ana positive,dsDNA and RA factor positive.anti ccp negative
a.cyclosprin
b.cyclophosphamide
c.hydroxychloroquine
d.NSAID
e.methotrexate

Patient who became agitated and irritable after sustaining head injury. Na+ =123 K+=4.8. What
is the reason for his symptoms?
a) SIADH
b) physiological response to injury
c) acute renal failure

A 55-year-old woman presented with discomfort in her both legs with an urge to move for last
4 months.She remains awake at night due to creeping and sometimes electric current-like
sensations.Her current medication include paracetamol, oxycodone and multivitamins.Blood
tests show normal renal function, normal electrolytes and slightly low haemoglobin.
What will you do next?
a. Cease oxycodone
b. Fluoxetine
c. Iron studies
d. Mirtazapine
e. Morphine
A patient was sent on a stess leave after employer finds him using cocaine. The patient admits
using cocaine and also has memory impairment, agitation and is also irritable, what is the next
step
a. urine drug screen
b. collateral history from employer
c. history of sexual abuse
d. past history of drug use

.A few weeks baby found dead in her cot ,she wasn’t cyanotic or any other distress , when
parents brought she already expired by than .what u do?
a.notify Coronal
b.order forensic autopsy
c.complete a death certificate stating as SID case

.an obese man with type 2 diabetes on metformin for 2 yrs, bp within normal range come for
f/u, lab test ; - hba1c (high than normal value ), glucose level high , wt to give ?
a. ramipril
b.insulin
c.gliclazid

40yrs male had fatigue, lethargy, joint pain, small testis, decrease libido, investigation?
a) testosterone
b) FSH
c)LH
e) iron study

History of young guy with bat bite what to give(NEW Q) A-antibiotic B-antirabies C-nothing D-
washareeba, 9:37 P
Ma lady who care about animals was bitten by a bat after a swab what is most important intial
a- wash the woundy
b- rabies immunization
c- antibiotics

a person who is diabetic takes insulin, At night before dinner,he takes short acting formulation
and before going to sleep he takes intermediate acting formulation,in the morning he takes
short acting insulin before breakfast,for few days he has been feeling dizzy in the morning when
he wakes up,what should be done initially?
-decrease the dose of morning insulin
-decrease the dose of short acting insulin at night
-decrease the dose of intermediate acting insulin at night
-stop insulin and start oral hypoglycemics
-refer him to dietician
A Somalian refugee with 3 month faecal incontinence has fistula now. What is the most likely
diagnosis?
Crohn disease
TB
Schistosomiasis –bowel schistosomiasis
Anal abscess

recall from a friend(13/2/18) Father with Huntington chorea, daughter carrier, she wants to
check her 10-year-old daughter?
1. Don’t do genetic testing –genetic testing not done under 18 years,HD is autosomal dominant
2. Counselling the 10-year-old daughter
3. Do the test for the 10-year-old daughter with her mother
4. Do genetic test for the daughter
5. Genetic counseling

Which test to check prevalence? Cross sectional

80 kg man came with 15 % burn how much IVF needed


A. 3L hartman +2 L 5 % D/W
B. 2L N/S +2 L 5% D/W
C.3 L hartman + 2L blood
D. 3L NS + 1L hartman
Parkland formula of burns= 4ml x %of burns(15)xBody weight = Quantity
http://www.vicburns.org.au/severe-burns/early-management/fluid-resuscitation/

list of drugs like metformin, digoxin , salbutamol puff and ibuprofen given and what to stop
prior to surgery- Nsaid

6. A medical student under the university supervision in your unit uploaded a paients
documents in Facebook, a social worker seen this and inform you what you will do?
A. this is not your responsibility
B. Warn the medical student
C. Inform the hospital manager
D. Inform to the supervisor
E. Ask the social worker to talk to the student

The pregnant comes with her husband due to (some pain or injury, I forgot). When asked the
history the wife said that she got those injuries herself from fall. When asked further the
woman seems withdrawn and anxious. On examination there’s also injuries on the vulva area.
What is the most likely diagnosis of this condition?
Domestic violence.
Others options I don’t remember.
The pregnant woman comes with her husband due to (some pain or injury, I forgot). When
asked the history the wife said that she got those injuries herself from fall. When asked further
the woman seems withdrawn and anxious. On examination there’s also injuries on the vulva
area. What would you do first for this condition?
Confront the husband about the injuries.
Admit her immediately
Ask her to come alone again
Tell husband to be gentle with her

Intern at the hospital. He knows about all the cases of needle stick injury in the. All cases in the
hospital were manage correctly with no big problem. Now there is accreditation and the intern
was asked about this issue. What the intern should answer:
1. Give info about the protocol of management
2. Leave the answer to your supervisor
3. Tell a case that he knows
4. Say that there’s no problem with needle stick injuries in this hospital

23 yr old lady came with weakness on all limbs but especially at lower limbs,diminished reflex (
not sure ) tingling sensation at foot, no other sensory symptoms .
HB reduced
MCV – 100 ( normal 80- 100 )
WBC, platelet – normal
B12 level- reduced
Likely cause ?
Pernicious anamia ( sorry forgot other opitions )

A woman works at a part-time job, and complaints of early morning headache, frontal &
bilateral, dull in character, varying in intensity, she takes paracetamol & ibuprofen, which only
cause relief for 2-3 hours. What is the cause?
A. Drug rebound headache-present all day everyday,,throughout the day
B. Migraine
C. Tension headache-bcoz early morning
D. Cerebral tumor
E. Pre-menstrual headache

Patient who is a known diabetic and hypertensive currently on ramipril and metformin for the
past 2yrs. 2 weeks ago she developed an upper respiratory tract infection and took amoxicillin
for three days and some vitamin c tablets. Now developed tongue swelling and bilateral
wheezing on chest auscultation. Cause is the most likely dug which can cause this presentation
A. Amoxicillin
B. Ramipril-bcoz amox usually has skin rash and ramipril …acei is common cause of angioedema
C. Metformin
D. Vitamin C tabs
What’s the most important marker in finding health risk of obesity? 
a.
BMI
b. waist circumference
c. waist hip ratio jm 891
Man who has claudication,low ABI ,has hypertension and DM , but not need to do surgery now,
what to advice him for his claudication
supervised exercise
reduced BP
lower cholesterol
Reduce smoking ( not stop)

A 49 yo man presents to your clinic because of fatigue for the past 4 months. He been having
trouble sleeping because of night sweating and has lost 5 kg without dieting; he also complains
of blurred vision. On examination he is pale and thin, with multiple ecchymosis, has a systolic
flow murmur (II/VI) and an enlarged spleen. Labs:
WBC: 95,000/mm3
15% blasts Haemoglobin: 7.4
15% bands Platelets: 88,000/mm3
51% PMN
13% Lymphos
Which of the following is the next step in diagnosis?
Coagulation studies
Cytogenetic studies
Iron studies
No further studies needed for diagnosis
Bone marrow biopsy-CML

During CPR by two people, the ratio of cardiac compression to resuscitation is ?


A) 5:1
B) 3:2
C)30:2 for adults always 30:2 for children some sites say its 15:2 in case of two rescuers
D) 15: 2

A man presented for a sick certificate but presented with ataxia asking most important
investigation to do
a. ct scan of brain
b. LFT –suspecting hepatic encephalopathy a ns imp to see liver deterioration
c. INR
d. fasting blood sugar
e. full blood exam

Patient put facebook status for you that you didn't treat him well and no one should get
treatment from you.what will you do?
A. Ignore
B. Ask pt to come to you and discuss with you
C. Inform defence manager
D. Write to ask him to remove post
E. Write all the treatment you provided under same post

A 4 y old Male child is brought to the emergency department after ingestion of his
grandfather's pills 50 minutes ago.
Which one of the following if present, is not a contraindication to gastric lavage??
A. Coma.
B. Presence of the gag reflex.absence is a contraindication
C. Ingestion of alkali.
D. Ingestion of acids
E. Ingestion of petrochemicals

several ppl in a city has abdominal pain, nausea, vomiting. There is a recent work with open
coal seam. What to do next?
12. Inform health authority
2. Inform environmental protection authority
3.check CBC,blood biochemistry

A 83 old man with an elective splenectomy planned, which is the vaccine needed by him prior
to the surgery?
A) MMR
B) Influenza vaccine
C) pneumooccal vaccine
D) TT vaccine.
Vaccinations against S. pneumoniae, N. meningitidis, H. influenzae type b and influenza virus
are strongly recommended and should be administered at least 2 weeks before surgery in
elective cases or at least 2 weeks after the surgical intervention in emergency cases.

1 year old child diagnosed with bacterial meningitis was admitted to hospital.blood culture
taken and iv ceftriaxone started.after 1 day of admission child had a seizure of <1 min
duration.long labs were given sodium and bicarbonate low.all others were in normal
range.what is the cause of seizure?

a.SIADH-though there is no low bicarb in siadh but it is mostly ass with meningitis
b.dehydration-can be possible
c.adrenal failure-k should be mentioned and high

Mother complaint that 8 years old has abdominal pain and day time bed wetting ..started 3
months back aftr her step father moved in..teacher compl that girl is inappropriately affectinate
towards elders at school
1 call child protection
Involve her biological father
Take more history from teacher
Take history from mother alone

which is correct about myasthenia gravis?


A) it is diagnosed by MRI or CT
B) It is associated with small cell carcinoma
C) it is associated with autoimmune thyroiditis

confusion and disorientation 3rd postop day, SPO2 88. Alcohol.


Investigation
Alcohol level
Blood glucose level
ABG
Cxray
CTPA
20 Y O F DRUG ABUSE BUT STOPPED SAID IT IS A NEW LIFE WISH TO CONCEIVE AND WISH HER
BOY FRIND TO STOP DRUG ABUSE WT TO TELL HER?
DON’T CONCEIVE BEFORE 1 YEAR
CHECK FOR HIV AND HCV****
SEND BOTH FOR DRUG ABUSE REHABILTATION PROGRAM
patient came back from camping trip complaining of loose motion.she had rash around
ankle.cbc shows eosinophils cause Jm-138
a) entameba
b) giardiasis
c) strongyloides******(blood esinophilia)

A patient with rheumatoid arthritis is about to be started on Adalimumab (TNF Inhibitor).


Screening of the following should be initiated before starting this drug.
a. CMV Serology
b. Toxoplasmosis Serology
c. Quantiferon Gold*****(tb reactivation risk)
d. HIV serology
e. Hepatitis profile

Resp acidosis case PH:7.3 PO2 :50 , PCO2:45(N-38-42)...also present with convulsion ..asked
cause of convulsion
A:Hypoxia***
B:Hypercapnia
C: Lactic acidosis

physician with hep c comes to u for review, still practices like before even though he was
explicitly instructed to take precautionary measures. what will you do?
a) notify board**
b) tell his manager/hosp director
c) tell him to be careful

Pt become Hypertensive after dialysis(due to overdialysis). Which drug will you choose
A. Losartan*****
B. Amlodipine
C. Frusemide
D. Carvidelol

When evaluating a report of clinical trial, which one of the following is correct?
A. Control and treatment groups must be equivalent in size.***
B. if randomization is conducted properly, chance differences are inevitable
C. Inadequate sample size has been shown to produce true positives and true negatives.
D. results are invalid if the trial is of not double blind construction.m

Teacher complain of shouldn't not regular in his lessons , leave school , mother say he
go ipto school in time but he became aggressive against his young brother
A-separation anxiety
B-truancy****the action of staying away from school without good reason; absenteeism.
C-conduct disorder
A patient has pneumonia. His hip muscles become weak. CT shows a spinal stenosis. He is on
statins and multiple other drugs. On examination knee jerk is absent, lower limb power is 3/6,
absent dorsalis pedis pulses and sensation over thigh is loss. Next investigation?
A) Mri ****
Xray.
C) Arterial duplex
D) CK
E) LP

50yr old man visits to Bali & after he got back, developed fever with chills, mild jaundice,
diarrhea. On examination, hepatosplenomegaly present.What will be the fact least indicate
malaria?
a) Diarrhoea
b) Daily fever***-cyclical or tertian(hot,cold & sweating stage) fever in malaria
c) Absence of splenomegaly
d) Jaundice

child with infection gram +ve diplococci.which ab to select?


a -penicillin*** ( streptococcus)****
b-flucloxacillin-penicillin
c-ciprofloxacin-fluoroquinolone
D-azithromycin-macrolide

http://www.medicalcorps.org/pharmacy/AntibioticsofChoice.htm
Daughter death has made him depressed on sertraline and now confused 
Na 124 ,K
dec,bicarbonate increase,rft normal 
Stop sertraline 
Iv fluids*** 
Na tablets
But i doubt the memorization of the Question .. i choose A as there is no any other Option that
is appropriate here.
SSRIs could cause Hyponatremia in the start of the treament but will not cause hypokalemia or
Metabolic Alkalosis ((Increased HCQ3))
Also SSRIs could cause SIADH and again it will be Metabolic Acidosis not Alkalosis.
The above Picture of Metabolic alkalosis + Hypokalemia + Hyponatremia is completely going
with Diuretics use but there is no mentions of Diuretics!

Treatment then stop SSRIs for hyponatremia then Water restriction


Post MVA pt having pain in bilateral both side if chest on ct there was effusion on one side of
chest there was widening of mediastinum but trachea was mentioned as centrally placed
Option-
hemothorax
Pneumomediastinum
pulmonary contusion

A patient comes to you at the ER due to severe back pain. He has already taken 600 tablets of
paracetamol for the past 24 hours. He has been in the ER frequently for the past 3 months.
What will you do?
A. Liver profile
B. Give oxycodone
C. Admit for further investigation****
D. Check creatinine levels
E. Refer to specialist
Old lady 85 years old living alone, diagnosed as hypothyroidism she was started on thyroxine
replacement the dose now 75 mcg after 3 months TSH: 16,6 ( high) and throxine low wt to do:
A. Confirm her compliance to medication ****?( non compliance - normal tsh , low thyroxine)
B. Increase dose to 100 mcg
C. Repeat the labs
D. Send nurse to give her the dose

Superficial thrombophlebitis with tender thick cord with H/O previous DVT
• Analgesic and mobilization****-if not severe
bed rest with leg elevation –if severe
JM 755

Q.recurrent vaginal candidiasis + oral white lesion bleed on scraping lymphadenopathy --


Glucose test***
vaginal swab
hiv testing
85 year old man with MMSE 10/30 rectal bleeding for almost 500 ML no ability to talk or decide
for himself. His family is insist of full health care for him. what is your next action?
a.immediate transport to nearest hospital with ambulance
b.suggest them to take him to the nearest hospital
c.arrange transport to a tertiary hospital****
d.consult the family for the end life support
A 14-year-old girl comes to the ED with a diagnosis of acute appendicitis. Taking the medical
history she says that has run away from home 2 days ago. Who is the most suitable person to
make the consent for her surgery?
a. The patient.
b. The patient’s parents****.
c. The patient’s boyfriend.
d. A guardian appointed by the guardianship court.
e. The patient’s best friend.

80 year old female after hernia repair, 12 hours later becomes restless, confused and tried to
remove IV access. SpO2 88%, BP 90/60, PR 100, RR 30. X ray done (increased radiopacity in the
left side and same but less extent in the right side). What is the most appropriate next step in
management?
jm-1411
A-intubation-
B-IV heparin
C-IV ceftriaxone
D-diazepam ( if i/m midazolam)
E.O2 infusion****

homeless man comes to emergency department, agitated and insist to admit him and
threatening that he will jump in front of a car if you will not fulfill his wish. He has been taking
low price beer for 2 cans for the past 2 weeks. What is your next step?
A. Mental health Act (something like that) B. Disclosure ?? C. Give him IV Glucose D. Give him IV
thiamine E. Give him Diazepam****
Which of the following drug used to control post parandial glucose control
A)aspart
B)acarbase
C)metformin
D)pioglitazone
E)tolbutamide
64 yrs old who has morbid obesity and type 2 DM on insulin , he has CRF , IHD , BP=155\100 ,
BMI=34 , pt has a concern about his weight , which of the following is a suitable agent to add to
his insulin ?
A- Exenitide-contra indicated in renal cases
B- Gliclazide-causes wt gain
C-Linagliptin-usually given when many comorbids present
D-Metformin-jm 196 table-contraindicated in renal cardiac and hepatic cases
E-Pioglitazone-causes wt gain

ECG of heart block again, pt was on multiple drug. What drug can cause?
a. Metformin**
b. Enalapril
c. pioglitazone
a patient with diabetic foot ulcer 1cm oozing pus and erythema admitted to hospital. what is
the next most appropriate step in management?
a.hba1c
b.oral amoxiclave
c.iv ticarcillin
d.mri foot***
https://www.mja.com.au/journal/2012/197/4/australian-diabetes-foot-network-management-
diabetes-related-foot-ulceration

You are called to see the same patient 7 days postoperatively as he has become unwell and
pyrexial with a temperature of 39.0°C. The patient has generalized abdominal discomfort. The
abdomen is tender with generalized guarding and rebound. The chest is clear to auscultation.
The patient's catheter and epidural were removed 2 days ago. The most likely explanation for
the patient's pyrexia is-
A. Deep vein thrombosis
B. Infective exacerbation of chronic airways disease
C. Pulmonary embolus
D. Anastomotic leakage****
E. Pre-existing chest infection
17) You are called to the ward to review a 72-year-old man who is pyrexial at 38.0°C, 8 hours
following an anterior resection for rectal adenocarcinoma without defunctioning stoma. He is
asymptomatic and pain-free with an epidural.A urinary catheter inserted in theatre is draining
concentrated urine. He has a history of chronic airways disease controlled with inhalers. He has
no respiratory distress, but both lung bases sound quiet. The most likely explanation for the
patient's pyrexia is-
A. Epidural abscess
B. Systemic response to surgical trauma****
C. Basal atelectasis( 1st day fever- atelectasis)
D. Infective exacerbation of chronic airways disease
E. Urinary sepsis

old man 60 years old who is a heavy smoker for 40 years comes to you with a swelling in the
tonsillar region, O/E there is a mass in the tonsillar pouch, what is your diagnosis ?
a. Nasopharyngeal cancer
B- posterior Lipoma
C. Lymphoma - Rubbery .
d. laryngeal cancer
e. metastatic spread****

A young women has cholecystitis (diagnosis mentioned by details). You are discussing ttt
options with her. She is telling you that she wants you to refer her to a certain surgeon because
she read on his website that he is doing laproscopic cholecystectomy with the smaller and
fewer wounds. You know that he has high rate of complications in his patients. What should
you do?
- comply to patient wish. ***
- tell her she should have her stones broken down with laser.
- tell her that this surgeon is incompetent.
- refer her to another surgeon
- tell her she should have open cholecystectomy.
Post colectomy pt for 24 hours, now oliguria for 6 hours, NS been given at fixed rate fever 38.6 .
Also generalised abdominal tenderness NEXT inv to order-
Ct abdomen****
Usg kidney
Blood culture
Urea electrolyte
9 year old patient comes with his parents because of multiple times upper and lower
respiratory infections. Lab values are given. (Hb decrease, MCV decrease)
1. Red cell folate –megaloblastic anaemia-MCV increased
2. Serum B 12 level - megaloblastic anaemia-MCV increased
3. Serum ferritin ***-IDA-ferritin dec and tibc inc
4. Hb electrophoresis-thalassemia(AR) but manifests by 2 years of age itself
A young man MVA fractured his ribs and has minimal left pneumothorax,peritoneal lavage is
positive for blood and laparotomy is planned for interperoneal hemorrhage, whats the most
important thing to do
A-Assesing of nasogastrics
B-Incerting of intercostal drainage tube****
C-determine PaO2
D-insessing of central venous pressure line

.A Schizophrenic patient, BP 190/130. Diagnosed as malignant hypertension. 3 months ago he


came to BP 160/110 and 150/98 after a 5-mins rest. What is written in his medical record is that
he comes for follow up at a medical outreach team. What is the likely cause for his treatment
delay?
A. His BP at the first presentation is low.
B. Stigma
C. He has no Gp

severe retrostarnal chest pain with low bp dall on percussion he has HTN DM asking most
diagnostic test ?.
-1.non contrast ct –oesophageal rupture??
2.electrocardiogram
3.chest xray

25 years women comes swelling in groin .u/s shows deep inguinal ring occluded.phisical
examination normal . what is your next management
1.Open mash repaire
2.Laparoscopic hernia repair
3.Review when lump appear

..35 yrsfemale,visual problem from last 1 year,left eye 6/12 & right eye ??pale optic disc,no
cupping,afferent pupillary reflex absent,DX?
a.DM
b.glucoma (causes of pale optic disc, means atrophic optic nerve,includes MS, autoimmune,DM,
viral and bacterial infections, toxins, allergies, mathanol)
no optic neuritis/MS in options

a lady came with tiredness and confusion.lab values were given-ca high,phosphorus
normal.asked what next
a. repeat calcium after2 weeks
b.parathyroid hormone assay
c.bone scan

Pt comes with haematuria and this was the ct,asked what will increase the chance of DVT after
his operation
.
a.Nicotine stain of fingers –slight risk
b. Atrial fibrillation
c.. Bilateral varicose veins
d. BMI
e. Spider naevi on chest

An old man comes in with lightheadedness and dizziness. He has OA, HTN, DM and AF. He is
suffering from GERD and had an endoscopy that proved H. Pylori infection and received triple
therapy and is still on omeprazole 20 mg daily since. He is taking metformin, verapamil,
paracetamol and timolol eye drops. His BP 106/67. HR 47. What is the cause of his condition.
A- verapamil and omeprazole.
B- verapamil and timolol.
C- verapamil and metformin.
D- Paracetamol and omeprazole.

A patient came with confusion. Was on multiple drugs. Known case of DM and HTN. Serum
sodium was less. Glucose was 8. It was mentioned that urine specific gravity was normal. Asking
for cause of confusion?
Hyponatremia
Hyperosmolar syndrome **
According to the consensus statement published by the American Diabetes Association,
diagnostic features of HHS may include the following:[6][7]

Plasma glucose level >30 mmol/L (>600 mg/dL)


Serum osmolality >320 mOsm/kg
Profound dehydration, up to an average of 9L (and therefore substantial thirst (polydipsia))
Serum pH >7.30
Bicarbonate >15 mEq/L
Small ketonuria (~+ on dipstick) and absent-to-low ketonemia (< 3 mmol/L)
Some alteration in consciousnes
33 year old female, in 3rd trimester, came with an injury which occurred due to the use of
garden implements. Her last DTPa was at 26 years. What is the most appropriate thing to do?
No immunization
Vaccine now** ??
Vaccine after birth
Vaccine and immunoglobulin now
Vaccine at 36 years of age

Patient on multiple drugs. Now comes with complains of fatigue without pain. Don’t remember
the exact drugs or the options but might be like:
Hydrochlorothiazide and NSAIDs
Hydrochlorothiazide and Amilioride
55 year old patient, came with severe pain at L4 and L5. Have a history of weight loss 6kg in 2
months. The pain travels down to the legs. No neurological deficit present. Asking for the most
important cause for investigating this patient?
Radicular pain
First episode
No history of trauma or strain
Weight loss **

your in a rural area and 4 cases of trachoma come to you then you find out extra 20 how can
you treat acute indexes ?
a-hand washing
b-azithromycin hb 3.385
c-doxycyclin
d-don’t do anything
e-penicillin

patient with cellulitis, given Cefazolin. Got better, but after 4 days returned with fever and rash
as shown (handbook pic)
Dx? a. Delayed hypersensitivty
b. Toxic shock
c. Steven Johnson
alcoholic man, GI bleeding 4 episodes. pulse 120, bp 100/something and falls to 85/something
on standing, initial mx?
o neg blood, iv colloid, iv adrenaline
1 isotonic saline
2 plasma expander
3 blood transfusion
.A pregnant woman with bleeding pv has blood group 0-ve, how will you assess anti D dose?
A. Feto-maternal hemorrhage
B. Assesment of anti D ig
C. Coombs test
D. USG
E kell test-kell blood group

Alcoholic man presents to ED . He’s agitated and confused. He said he’s been drinking cheap
wine and eating foods from scrapes. What’s the next step in investigation?
- blood sugar level
- magnesium level??
Alcohol level
First thiamine then check bs give iv glucose then alc level,if sevry comatose or mentally altered
then alc
https://www.ahcmedia.com/articles/35285-management-of-the-intoxicated-patient-in-the-
emergency-department
old man h/o smoking , alcoholism.presented with dyspnea.O/E plethoric face, engorged neck
veins.what is most appropriate investigation? (sorry don’t remember full scenario)
1- Xray chest
2-ECHO
3CT CHEST-svc syndrome
Causes of Plethoric face
5 C's
Chronic alcoholism
Cushing's syndrome
Chronic cor pulmonale
Carcinoid syndrome
SVC syndrome

Old age smoker came with facial plethora and difficulty in respiration when lying down, history
of weight loss present. On examination, reduced breath sound in right upper chest.
CT chest-carcinoma lung
Venogram
Forget others. (Soe, Bo recall)
A woman after removal of central venous line lady developed facial swelling and swelling
around the neck. What’s the most appropriate investigation?
a. CT chest
b. CT neck angiogram
c. Chest x-ray
d. Neck USG Doppler
e. Echo

many scabies cases in children in a remote indigenous community.after treating patient what
most appropriate to do?
1-all children should stay away from school
2-close all swimming pools
3-treat all household contacts-permethrin or benzoyl benzoate, sulphur or crotamiton in kids
<2months

. a lady presents with lethargy low mood, amenorrhoea and bouts of constipation. on
examinationshe had an anterior neck swelling which moves with swallowing . lab TSH high T4
Low . you suspect hashimoto thyroiditis apart from starting thyroxine which of the following
will you go to confirm the diagnosis
A. thyroid ultrasound
B. FNAC
C. Antimicrosomal antibody test aka peroxidase antibody test
D. trial of thyroxine therapy starting dose 75mg
E. Anti peroxidase antibody test-
another very twisted and long drug interaction.. pt on aspirin, metoprolol, prindopril,
indopamide, & mixtard insulin. Suddenly fell and collapsed. On presentation had BP 90/80, RBS
of 5.5. I was stuck on this one. Asking what drug caused this? Options were:
a- Insulin
b- Aspirin
c- Metoprolol
d- Pendropril-Indopamide

Man came after chlamydial arthritis......got sulphasalazine....and now is having high fever,
painful mouth ulcer, on exam swollen gum and ulcer on gum and buccaneers mucosa found...
Lab showing neutropenia cause asked
1.sulphasalazine toxicity
2.Herpetic stomatitis-if fever present more likely infection

. A lady with CCF taking metformin, furosemide n some htn drug. Which one of the followings
should NOT be prescribed
A. Perindopril
B. Spirinolactone
C. Isosorbide nitrate
D. Diltiazem
E. Losartan

. A man with BMI 24, waist 76 cm,(N 102) sugar fasting 5.2, total cholesterol 5.5, what will lead
to further CVS risk
A. Obesity
B. Diabetes
C. Hypercholestremia

Middle age man WITH pain in buttock and thigh during 100 m walk on ground. He can walk 20
m uphill but his femoral pulses are not palpable however his dorsalis pedis is palpable. which
appropriate investigation will you request
A. Arterial Doppler
B. Digital subtraction arteriography
C. CT angiogram
d. femoral ultrasound scan
JM ABI: 0.9-1.3
venous ulcer: normal ABI>0.9
Arterial ulcer<0.5
Mixed: 0.5-0.9

A known type 2 diabetic man with history of af , now presents with painful and swollen leg for
12 hours, on examination he has a temperature of 38.2 and calf redness, what's your initial
investigation?
A. Blood culture.
B. Fasting blood sugar.
C. Doppler ultrasound of the affected leg. ( Complication of DM KNIVES)
D. Chest xray.
E. D-dimer.

. A man comes to you following an episode of rectal bleeding which has stopped spontaneously.
You are planning to do colonoscopy. He had a drug eluting stent placed 2 months ago and was
started on clopidogrel and aspirin). Which of the following is the appropriate mode of action?
�a. Stop clopidogrel and do colonoscopy
�b. Stop clop for 7 days, give LMWH and do colonoscopy
�c. continue clop and do colonoscopy ( since stent is present antiplatelet drugs should not be
stopped for 2 years post stent insertion thus continue and do procedure, if no stent present
then stop)

pt on stent and clopidogrel for four months and her elective surgery is planed for varicose
veins. Came for ur advice
What is ur most appropriate step?
Reascess for surgery
Stop clopidogrel and proceed for surgery
Give ffps and do surgery

old patients with MI and stent 2months ago. Now he is on clopidogrel now falls and has a
fracture of neck femur FBC and platelet morphology normal,INR-1.5, but has bruises on the
body what should be done?
A. stop clopidogrel immediately and do surgery in 1 week if there is no stent—if it was not
urgent then this option
B. give FFP and do sergury now
C. give platelet and do surgery now
d, do surgery now-as its urgent surgery
a man with history of limb claudication on 100 meters relieved by rest, on examinations there
was absent left femoral pulse and absent dorsalis pedis pulse , ABI done and it was 0.24 . what
is the most appropriate test leading you to the diagnosis? ‫؟؟؟؟؟؟؟؟‬
A-Arteriography (ABI=Ankle brachial index normal ratio=1)
B-Ct angiography
C-compression Doppler ultrasound
D-MRI
E-X-ray

a mother of two children healthy presented to you asking you to have test for
hemochromatosis for her and children as her brother diagnosed with this disease. What is your
most appropriate advice?
A-iron blood test for children
B-do test for mother alone ( scening is done by checkin serum transferritin level or serum
ferritin level in parents,siblings and children of affected/diagnosed individual, since here the
brother is affected only sibling i.e mother in this case should be checked)
C-refer her and children for DNA test
D-ask for an appointment with the husband and the mother

obesity most imp indicator of health risk


A. Bmi (Others may be more accurate but still BMI first)
B.waist circumference
C.waist hip ratio=cvs risk

27. what is the best measurement for health risk in obesity?


a. BMI***JM 71
b. waist circumference( metabolic syndrome)
c. waist : hip ratio male less than 0.9 and for female 0.8 calculate in CVS risk JM 71

HSP scenario with photo, develop rash in buttock and leg, next investigation
A) abdominal USG
B) urinary phase contrast microscopy (urine analysis is most imp test for HSP)
C) blood culture
D) cxr
HSP-HAARPh-hsp, a-arthritis, a-abd discomfort, r- renal manifestations-
htn,proteinuria,hematuria, p-purpura
Inx-urine analysis and microscopy and proteinuria,hematuria-only imp assessment
Rx: NSAID & CS, self resolving
which of the following has the largest ionizing radiation per capita
1. nuclear implants
2. diagnostic xrays
3. radiation from electronic devices

colleague taking marijuana during the time between the consultations with patients, who to
do?
1. advice him to stop taking marijura
2. notify medical board
3. notify senior

. man brought to rural clinic after bitten by snake at the back of beer pub. He said there was a
long brown snake but his friends couldn’t find the snake. On examination, there was 5 cm long
scratch on his leg but no bitemark. What to do?
1. take skin swab from wound and apply pressure dressing
2. reassure him as there is no bite mark
3. apply tourniquette
4. giveantivenom
you are a doctor in rural hospital in north Australia and a patient comes after 30 mints was
bitten by a brown snake in his left ankle , the patient has no symptoms , and there were simple
scratches over the skin of the ankle with no marks of the snake teeth , the tertiary hospital is
150 km far away , what is the urgent next step to do:
A- Call for helicopter ambulance to the tertiary hospital to be supervised
B- Give anti- venom ampoule now and another ampoule after symptoms appear
C- Tell the patient to go home as no tooth bits remarks and no symptoms
D- Urgent apply for a tourniquet in the upper part of left thigh.
E- No option mentioned to manage the scratches locally or bandage locally

question of Rheumatic fever asking for inv

33 year old lady comes with lump in the left side of neck for 3 months which is 3 cm size
papillary carcinoma with no metastatic spread. What is your most app management.
1. total thyroidectomy
2. left thyroid lobectomy
3. radio I 131 ablation
4. radio I 131 ablation followed by thyroidectomy
5. suppression with thyroxine
Surgery is the definitive management of papillary thyroid cancer. Approximately 4-6 weeks
after surgical thyroid removal, patients may have radioiodine therapy to detect and destroy any
metastasis and residual tissue in the thyroid.
External beam radiotherapy has been used as adjuvant therapy in patients with papillary
thyroid cancer who were older than 45 years and had locally invasive disease
man with history of angina. Now becoming more frequent angina. Smoke 40 cigars per day and
drink 3 glass of wine in the evening. Body weight – normal. What next?
1. assess his intention on changing life styles.
2. tell him he must stop smoking
3. stop drinking alcohol
4. lose weight

scenario of a man was in confined place rushed by ambulance to the hospital emergency now
ask how to give o2.??
A.2l nasal.
B. 6l Ventura mask.
C.only give if spo2 less than 90
D.immediatly enter him into hyperbaric oxygen room
E.continous high flow o2mask(don't exactly remember the last opt.but similar to this)

26yo male. Forklift driver working in a confined space, having headache, lightheadedness,
nausea and abdominal cramps. What to do next?
A. Check O2 saturation before giving O2.
B. Hyperbaric chamber O2 therapy.
C. Nasal prong O2 3L.
D. High flow O2 mask

patient known diabetic, hypertension on ramipril, amiodarone, statin, cxr shows diffuse
pulmonary marking, oedema,what is the cause
1. statin
2. amiodarone (causes pulmonary side effects-fibrosis)
3. ramipril

A scenario of interstitial lung disease. CXR given (image as below).


Cause – Amiodarone

Cerebellar signs bilateral worse on left (diplopia Ataxia nystagmus) he was on carmazpamine
and then started on erthrmoycin, lateral gaze
PICA-only one side-ipsilateral
Left cerebellar stroke –only one side-ipsilaateral
Carbamzipine toxicity--Mostly cns S/E, charcoal hemoperfusion

Silvariy gland choronic parotitis lump at angle of mouth mobile painless and FNAC done
MRI
USG
Plain of the mouth
Sialogram
If ct given chose that

Hutinchisen present since 8 years slowly growing what will you do


No cure, treat symptomatically,refer to support groups, may progress to dementia so can give
halo…JM

Post op Ozygen saturation is 88. drinks 14 cans of alohol daily, no fever, agitated
No option of ABG
Chest xray ??????( since it is post op case and pt is showin low 02 level and agitation think
hypoxia think pul embolism, for which steps are first pulse oximetry then ABG then CXR)
Check blood acohol (levels alcohol causes decreased 02 saturation.)

32 year old woman presented to you asking your advice .she told you that in her new job she
works at night shift in most days and she worried if she will have vitamin D deficiency ,
previously she was a very active person and sport in solar days . what is your most appropriate
action?
A-prescribe her oral vitamin D
B-measure her vitamin D level
C-told her that vitamin D not taken from sun only but also from food
D-tell her not to work at night
E-ask her to eat vitamin D rich food
Old man with a fracture between T8 and T10 showed increased uptake on x ray. Labs were
given with anemia and very high ESR. What is the next investigation?

Plasma electrophoresis bcos looks like mm


PSA
In MM bone lesions are lytic in xray but in metastasis of prostate cancer its sclerotic !!
If the increased uptake was only defined to fracture site then A
but increase uptake means sclerotic ...so i m with prostat cancer

Old man with fracture of T8-T10….labs value were given..low HB and MCV..raised ESR > 100.
whatdx
PSA-since esr inc
Bone marrow examination-MM here as no xray given & no urinary symptoms
Blood culture

Axillary lymphadenopathy with blast cells 66 per


Cll-mainly lymphocytes
Acute leukemia-mainly lymphoblasta
Lekemoid reaction
Hodgkins lymphoma-mainly lymphocytes

sjogren scenario, dry eyes, dry mouth, mass in mandible fixed. Dx test
Ana
FNAB
CT

Sjogrens diagnosis
1)ANA
2)RF,RO-SSA,LA-SSB
3)ESR
4)immmunoglobulins
5)schirmers test for opthal feats-rose bengal dye…
6) salivary gland biopsy from lip
Rx symptomatic,no cure
NSAIDs-naproxen,later steroids,hydroxychloroquine-systemic sx

HB question, Gram negative septicemia after instrumentation


Hereditary spherocytosis inheritance  50% in all siblings
DKA scenario, how to follow up  HBA1C
Scenario of Tourett disorder can be genetic,2-20 yrs, simple –tics
,grimacing,shrugging,blinking,grunting , complex-jumping spinning,touching, vocal, ass with
ADHD,OCD,CD, Rx cbt-habit reversal training , Fluphenazine, haloperidol,botulinum toxin inj,
ADHD medications- methylphenidate, antidepressants, Antiseizure medications.
Scenario of Reactive attachment disorder- a condition found in children who may have received
grossly negligent care and do not form a healthy emotional attachment with their primary
caregivers -- usually their mothers -- before age 5
Dependent personality disorder (woman with 2 previous marriages, live with mother)
What to monitor Guillaine-Barre inx-LP,EMG, FVC
Rx-plasmapheresis, IVIG,resolves in 6-12 mnths

A 62 yr old male Non-smoker <diabetic>with bp of 125/89, total cholesterol and HDL ratio was
6.2 was asking you regarding the risk of cardivascular diaeases in future ??
37 / 39
10-15%
16-19%**** (60> with DM is always considered high risk regardless of their calculated risk)

In accordance with Australian guidelines, patients with systolic blood pressure ≥180 mm Hg, or
a total cholesterol of >7.5 mmol/L, should be considered at clinically determined high absolute
risk of CVD.
Adults over the age of 60 with diabetes are equivalent to high risk (>15%), regardless of their
calculated risk level. Nevertheless, reductions in risk factors in this age group can still lower
overall absolute risk
last 2years,on examination there was red,painful, lumpy lesion on leg with bilateral ankle
swelling, no pic was there,serum ACEI was high,was asking wat will you do next
ANA
Skin biopsy –Dx is sarcoidosis(granulomas form) in which ACEI high ,painful lumps and Dx is
CXR,CT,MRI,Biopsies of lesions,Rx- no cure,corticosteroids, immunosuppressants,
hydroxychloroquine,tnfi.
Blood culture
A postman got ankle injury following leisure play,there was no abnormality on xray but dr
adviced not to go work for a wk. he requested to give him compensation certificate as he has
only 2days leave remaining,what will you do?
Give him 2days compensation certificate and 3days sick leave certificates
Give compensation certificate for a week
Give sick leave certificate for a week
Tell him to mobilize as early
Compensation is when it is a work related injury or illness.

Almost similar pic but it was on medial malleolus with pus in it,pt was diabetic,was on
metformin, his glycemic control was good for last few years but now its not in control. What
will you do next, no duration was mentioned—
Xray
Mri
Blood sugar
Swab from ulcer

Indigenous 15 year with Flitting joint pain,pain was changing its site when one gets better and
there was rash in back with H/O sore throat a week ago wat will be your next inv--
a.ESR
b.Throat swab.
c.ECG
d.Full blood count
e.CXR
There was q of gastroenteritis in party where multiple options given as cause
With people with symptoms and without symptoms
Berry shake
Egg
Salad-raw veggies cause gastro
Crab
Ham

pic of dupyetrens contracture to me it was grade 3, scenario was 63yrs old farmer complaining
difficulty to do works due to his hand deformity,he is smoker,drinks 4cans of beer a day, he had
cut injury to this same finger one yr back,was delayed in healing,wat is the cause of it?
Alcohol
Smoking
Because of farmer
Cut injury

A lady with BMI of 35 how will u manage her in addition to exercise,weight reduction and diet?
Involve in sports
Low carb food always
Lipase inhibitor
Diuretics
Surgery
lIthium poisoning>3 with associated feature ..Most appropriate management is what?
Himo dialosis –if toxicity featurews
0.9% normal saline-if no toxicity feats
Antidote for TCA

if the person comes with in one hour of ingestion so we can do gastric lavage otherwise fluid
therapy if lithium level is less than 3 than normal saline therapy
If lithium level is more than 3 hemodialysis and if there are symptoms of convulsion or comma
than again do the Dialysis .. if patient has liver kidney and congestive cardiac failure than again
do Dialysis

schizoaffective disorder on risperidone, and lithium. Went to party n took alcohol, cocaine n
ecstasy
Now presenting with hyperreflexia, tremors, irritability, drowsiness, agitation, temp 37.8, bp
150/?
Wat nxt investigation
Alcohol level, (hypothermia,hypoglycaemia,seizures,vomiting,confusion)
drug screen,
lithium level,
ck level

Ph 7.32, PCO2 normal, PaO2 decrease


Respiratory acidosis with no compensation
Metabolic acidosis with compensation
Respiritory acidosis with compensation

Patient with DM controlled Haemachromatosis, which will be initial investigation:


a) Iron study(serum transferrin saturation and sr ferritin)both increased,next do genetic testic-
c282y and h63d
next is MRI or CT
best biopsy
Patient with known Waldenström's macroglobulinemia is admitted with fever. He has a history
of glandular fever infection and varicella infection in childhood. He is given ampicillin and
azythromycin. The next day he develops rash. Picture: rash is on the abdomen and chest. It
doesn’t look like erythema multiforme at all. Most of the elements are raised papules, but
couples of them are pustules. What is the initial next step to identify cause of rash?
a. swab from the lesion??
b. biopsy of the lesion--since rash is not itchy or painful cant be varicella related
c. bone marrow biopsy
d. drug allergy test
https://www.dermnetnz.org/topics/waldenstroem-macroglobulinaemia/

Waldenstrom macroglobulinemia case presented with features of pneumonia admitted in


hospital and started on ampicillin and azithromycin now developed generalised vesicular rash
Immunoglobulin
Cease ampicillin and start ceftriaxone
Prednisolone****
Ganciclovir
Amphotericin
if scenario ask about investigation and than ans would b skin biopsy ..

)Child presents with DKA. What is the most important side effect that can occur during
treatment?
Cerebral edema***
Hypoglycemia
Metabolic acidosis
hypokalemia

Laparoscopy done due to some reason i forgot.... after some days fever and redness on
umbilicus... swab taken blood culture done. Result says staphylococcus. What is the source of
infection?
Nosocomial
Umbilicus

Nck lump without pic. Diffuse mass on thyroid.hyperthyroid feature with fine tremor and so on.
Raiu done.. low uptake.dx?
Graves disease
Toxic multinodular goiter
Toxic adenoma
All above conditions it is increased uptake….it is decreased in de quirvans
thyroiditis(painful,initially hyper layer hypo,self limiting) and factitious thyroiditis.(caused due
to excessive medication for hyperthyroidism, usually no neck lump)

Dm pt glucose uncontroled now, peripheral pulses are normal, medial malleolus non healing
ulcer with pus, after taking swab and blood for culture what next
Xray
Angiography
Doppler
Best option is MRI to check underlying vasculature in dm ulcers.

Rta q. Iv colloid given twice still in shock. What is most appropriate next
Laparotomy
Fast usg
Ct abd

. Medical practitioner has hcv ab and hcv rna positive 1 yr ago, now comes for insurance check
up. What will examining doctor do?
Inform medical board
No need to inform because of pt confidentiality
No need to inform if he agrees to modify his practice
Inform his hospital authority

Need to inform dept of health within 5 days

. Girl with that scar marks on wrist recall... she says not to tell parents, comes with sister, lives
with mom but wants to liv with dad... both parents and school counsellor called doctor.whom
to tell
Mom
Dad
Sister
Both parent
School counselor

35 yrs Man with malaise fever tiredness, has 6 month old boy. Pbf shows atypical lymphocytes
asking dx
Acute leukemia
Cmv infection
Hodgkin disease
Looks mostly like EBV Mono but if option not present go for CMV.

Anaemia scenerio with iron reduced, ferritin raised, tibc reduced. Asked cause
Iron deficiency anaemia-tibc inc,ferritin and iron red
Chronic inflammation

Multi drugs dm pt with warfarin....h/o af,added aiodarone 2 weeks ago... now thigh 4cm
enlarged than other one, irregular pulse, temp 37.8
Cellulitis
hematoma
Dvt
AMIODARONE increases the INR by interacting with warfarin,those pt's who has to be given
amiodarone needs dose reduction of warfarin.so looks like he developed hematoma after
starting amidarone.
Middle age man with HTN, DM was on warfarin,perindopril,statin,metformin.1wk ago he
started to take amiodarone, now present with right thigh swelling. On examination temp 37.9,
tender, red thigh. His Rt thigh is 4 cm bigger than left in circumference. what's your dx? (no
heamatoma in the option)
1.aterial embolus
2.Cellulitis
3.Dvt
4.Drug INTERACTION

Post carotid endarterectomy pt went home after 7 days came back with audible stridor and
severe dyspnoea , wut will u do after u raise his bed feet?.
A) immediate Intubation
😎 O2
C) Remove suture
D) Send to back to OR and surgery immediately

father and mother came to you they both have thalassemia minor traits in their genes what is
the possibility of their children to have the same thalassemia minor traits
zero
25%
50%
67%
100 %

Which is correct except-


Promiscuity with ca cx
Ebv with lymphoma
Nickel with hepatocellular ca-causes lung ca
Bettel nut with oral ca
Patient presents with cough productive of blood stained sputum. He has been having ankle
pains and painful wrist. Urine microscopy showed RBC casts. What is the diagnosis?
a. Systemic lupus erythromatosis
b. Bronchial cancer
c. Wegner’s granulomatiosis****
d. Renal cell carcinoma
Rx: corticosteroids
10% burn pt gets angry and wants discharge recall
burns pt was admitted for treatment. He had 10% burns. He overheard you talking to nurse and
did not like what you said and he wanted to go home
a) allow him and to leave and ask for him to come as outpatient followup
b) talk to him
c) call supervisor
d) call burn register and tell that pt wants to go

female lost job, crying, lost money in gambling, crying to buy food for children
arrange social welfare
inform child protection service
refer gambler something

which drug shud be stopped at least 7 days before operation


Amitriptyline

which one less effective


Case report
Cc
Cohort

Absolute risk of a disease is 6 in 100 ppl....relative risk for the dis is increased 50 % in smokers...a
new med is introduced which will lower therisk by one third for smoker people...what is
absolute risk with new medicine...
A. 6%
B. 12%
C. 33%
D. 2%
On methotrexate and prednisolone complaining of mouth & tongue ulcer. Treatment asked
a) Folic acid -if MXT toxicity not occurred and rx still goin on and want to give precaution then
chose folic acid.

b) Folinic acid-if MXT toxicity has occurred(mouth and tongue ulcers here) give follinic acid,if MXT
toxicity not occurred and rx still goin on and want to give precaution then chose folic acid.
c) Stop methotrexate
d) Increase prednisolone there is bone marrow involvement

)patient,labs showing megaloblastic anemia,has numbness and paresthesia,difficulty walking at


night,she is not strict vegetarian...indicates deficiency of,options
folic acid
,vit b12
a lady with anemia mild icterus NOT vegeterian NO menorrhagia..macrocytic megaloblastic pic next
step
red cell folate
vit B12 first b12 then if inc do folic acid
intrinsic factort antibody
iron studies
https://www.aafp.org/afp/2009/0201/p203.html

Patient comes back from Thailand with hx of having sex with multiple prostitutes. After one week
presents with lymphadenopathy, mild pharyngeal erythema, splenomegaly, sore throat
Atypical lymphocytes present. Monospot test negative. What is the most likely diagnosis?
a) HIV-seroconversive illness that happens 1-6 weeks after getting infected,aft this agn
asymptomatic for 3months(min) or years
b) EBV -can still be present evn if mono test is -ve
c) CMV

Old man came with his wife.he gets confuse sometimes and forgetful. He still drives, sometimes at
night .mmse 26/30 .what are you gonna do about it?
a- Cease driving at night
b- Repeat mmse after 3 months
c- MRi of brain
d- encourage more social and physical activities (MMSE) score of ≤24 and aggressive or impulsive
personality characteristics
78 years old man brought by wife. Wife is complaining that he cannot drive properly at night.
You checked MMSE 26/30. no nerological deficit. What would be your next management?
MRI brain
Review MMSE after 3 months
Cease driving at night.- These included the clinical dementia rating score, a carer’s rating of a
patient’s driving ability as marginal or unsafe, a history of reported traffic offences, a history of
crashes, reduced driving mileage, self reported situational avoidance, Mini- Mental State
Examination (MMSE) score of ≤24 and aggressive or impulsive personality characteristics.
https://www.racgp.org.au/afp/2012/april/dementia-and-driving/

Secondary school student with asthma planning for a school trip. Her teacher calls you to ask
updated Rx plan for her asthma because she couldn’t contact student’s parents. Your contact
and name was in student’s record book as student’s doctor. What to do?
Ask her to bring student quickly
Fax Rx plan to the teacher-if leaving immediately
Contact parents to get consent

Needle stick injury, known HIV pt. What next?


Restrict the Dr from practice
Antiretroviral as soon
Blood test for infection
Drug company introducing a drug to replace Aspirin. Got already approvement from the board.
They says it has no bleeding risk. Asking about what should we concern?
Common side effects
Drug interaction
Number of Pt need

A man after returning from Thailand 5day trip…hving fever, headache, muscle pain. Dx?
Malaria
Dengue

JM-widespread in southest pacific and endemic in Queensland .a returned traveler with fever
and myalgia is more likely dengue than malaria.rash also +

bipolar patient on lithium and controlled, now has tremor of hand mild at rest, increasing with
activity
A ..lithium level
B.. Change to valproic acid
C.. Add propranolol
D.. add benztropine

A 16 year old girl comes to you for screening, what will you screen for at this moment?
a) Scoliosis-age 11-13-more common in girls- Minor curves <25 degrees do not require any
treatment and observation only is indicated throughout the growth phase. –casting, bracing,
growing rods
b) Skin cancer
c) Breast cancer
d) Depression
go for ca cx if given

5 year old boy falls from a tree, he presents with a few scratches and cuts and no mention of
dirty wound.
Tetanus + topical Antiobotics
Tetanus and booster in 2 months
TT and Ig
( I’m sure you are familiar with the other options)

Note: Pay attention to the choices, I picked A because B didn’t seem right and he had multiple
scratches and cuts it seemt like the better choice
Antibiotic prophylaxis is not indicated for the prevention of tetanus; however, the use of
antibiotics (such as penicillin, amoxycillin + clavulanate, or metronidazole) for preventing other
bacterial infection of the wound is a matter for clinical judgment.
Ideally in this option no need for tt
http://www.immunise.health.gov.au/internet/immunise/publishing.nsf/Content/Handbook10-
home~handbook10part4~handbook10-4-19

A 56 year old farmer from queensland presented with a lacerated wound from a farming
equipment.he had similar wound 5 weeks ago and received tetanus toxoid..his tetanus immunization
schedule is complete..what is the next most appropriate mx?
A.start penicillin
B.give TIG
C.Give ADT vaccine ans to protect against diphtheria
d.Give TT and TIg
e.discharge home with follow up to consider antibiotics

A young doctor has decided to study the cause of dementia. He selects 190 men and women with
dementia and 190 men and women without dementia and reviews their previous histories looking
for factors that could determine the cause of dementia. This kind of study is called?

a) Case control –will frst select the disease and then see cause
b) Cohort –will select specific cause and compare
c) Cross-Sectional
d) Case study
e) Randomised control Trials
. A study wish to make a relation btw the fatigue in track car driver in high ways & the incidence
of MVA happen. wt is the most suitable method to carry out study?
a, cohort-cause to effect-hasn’t happened
b, case control –bcos from effect to cause-alrdy happened
c, RCT
D.cross sectional

3. Appropriate step in designing a cohort study regarding the Psychological changes in the
group affected by bush fire
A. Select a proper random numbers
B. design a proper efficient followup
C. Design a control group
A patient who is Diabetic underwent a sugery for appendicitis (can’t really remember which
surgery) and is also on thyroxine for his goiter. He now presents with weight loss and
tachycardia, his levels were checked 3 months ago and they were normal, what to check now?

TSH
FBS

An elderly woman on polypharmacy GFR was 6 weeks back came with dyspepsia, was put on
Olazpine and anatacids and is now presenting with confusion, loss of appetitie and nausea.
Sodium 129, k 5.3, calcium 3, phosphorus high , creatinine high, urea very high and bicarbonate 20,
what is the reason for the confusion?
a) Uremia
b) hyponatremia Neurological symptoms typically occur with very low levels of plasma sodium
(usually <115 mmol/L)
c) hyperk
d) hyper ca
e) metabolic acidosis
Patient on throxine + carbamezapine, now feeling weak, his labs were Na = 112 mmol What will
you do?

a) Stop carbamezapine + hypertonic saline


b) Stop throxine
c) Stop carbamezapine restrict fluid
d) Stop another drug
e) NS

Man has a history of angina, as well as DVT, pulse absent and is a heavy smoker and drinks
alcohol daily. what will is the cause of his current health problem? I felt like this was a PAD
senario and they wanted to test us on whether we knew the appropriate risk factor
smoking
Alochol
DVT
(I’m unable to recall the rest of the options)

2yrs old kid, drinks from guests drinks, now confused, drowsiness, what to check now?
check blood glucose-The reason is that Alcohol can make blood glucose level of young children
drop down to a very dangerous level. If it's not checked and not taken care of, the child can die
b) drug screen,
c) ABG

Salmonella outbreak has been declared in community. A girl present to you with watery diarrhea
for 3 days. Stool culture negative.what is the best test out of the following to dx her condition?

A. Blood culture
B. Stool culture Multiple negative stool cultures are required to confidently exclude ongoing
excretion.
C. Urine culture
D. Throat swab
Mother living in mining town comes with her daughter for normal blood test. Her daughter’s
test show lead level 0.72(14ug) what will be your next appropriate step?

a) Educate mother about environmental risks- n children: Blood lead level of 5 µg/dL or 0.24
µmol/L or greater requires no testing and action. The source of lead must be found and
removed. A lead level greater 10ug/dl (0.48)requires testin ig,development etc. A lead level
greater than 45 µg/dL or 2.17 µmol/L in a child's blood usually indicates the need for treatment.
b) Assess IQ testing of child
Refer for chelation therapy
d) Move the family to another town
e) Urine test to confirm dx

http://www.sahealth.sa.gov.au/wps/wcm/connect/42f4440048734e50bcdffd48c28c225a/NHM
RC+Lead+Level+Guidelines+FAQs+FINAL%5B1%5D.pdf?MOD=AJPERES&CACHEID=42f44400487
34e50bcdffd48c28c225a

Human bite from a stranger with bleeding and teeth mark. TT taken recently. What to give next?

a) Hepatitis B immunoglobulin and hepatitis B vaccine


b) Zidovudine
c) TT immunoglobulin
d) Azithromycin-penicillins,ceftriaxone,amox,metro etc but not azithromycin
e) Suture and observe after 24 hour

A 21 year old female presents with epistaxis, her lab results reveal a Hb of 8.5 gm/dl, a Ca level of
1.9 (normal is 2.1), INR of 1.5, what would be your management?

A. IV vitamin K.-dx- celiac disease


B. Oral vitamin K.
C. IV Calcium Gluconate.
D. FFP.
E. Blood transfusion
celiac disease fat soluble vit are deficient like A,D,E and K...... as a result of that def inr is
high....that lead to the bleeding....so option A

A 53 year old woman has not been sceened for the past 5 years. Sister diagnosed with colon cancer
at 63 years. Pap done 3 mths back and normal. Father and mother died of heart disease at 65 years,
what screening will you do for her now?

a. Colonoscopy actually should be fobt if given in options


b. stress ECG
c. Pap smear
d. Ultrasound breast

52 year old woman to whom Pap test performed before 2 years and it was normal on examination
she is healthy woman with no evidence of family history. Apart from Pap smear. which is the most
important screening test to advise her at this age?
a) mammography
b) glaucoma screening
c) colonoscopy
d) chlamydia test
e) other non-relevant

Child present with urine ketone, glycosuria, dehydration but urinate 4 times since morning.
During resuscitation what most serious complication can developed.
A. Hypokalamia
B. Metabolic Acidosis
C. Cerebral oedema-most serious
Most common-hypoglycemia

Hereditary spherocytosis scenario of a lady with blood picture which shows Hb 8.9 and 0.05%
retic count. Rest of the cells are within normal range. Which is the most likely organism for her
condition?
a. Adenovirus
b. Parvovirus
Man brought to the rural clinic after he was bitten by a snake through his trouser. There are
two bite marks on his leg. Which of the following is the most appropriate management?
a. Give antivenom
b. Apply torniquete and leave it for 2 hours-could have chosen this if no leaving mentioned
c. send the patient home as there are no signs and symptoms
d. Tell the patient that it is more likely to be a non-venomous snake bite
e. careful observationand giveantivenom wen signs develop

)Old woman known to the practice comes to you and she wants to change her will. Her
daughter is against it saying her mom has memory issues. On exam, the old woman is fine and
memory condition same as before (just mild deficit, no MMSE given) Asked next.
- tell her that she can change will
-refer for neuropsychiatric assessment
-she cant’ change will
-suggest attornery or lawyer something consultation

Question on Vitamin C efficacy on influenza with placebo, what to do to increase validity?


A) Random sampling –chosen randomly so that each person has a chance of being selected
b) Randomisation –rct-pt randomly allocated to each group
c) increase study power
actually best is blind study but if not present go for randomization
50 year old male comes for routine checkup. He does not have any significant medical history
however; he has couple of high BP recordings. On examination his BP I s130/78 but it has been
fluctuating between 130 and 175 systolic. What is the best recommendation for this patient ?
a. Holter monitoring
b. 24 hr ambulatory BP monitoring-
c. Stress ECHO
d. Tread mill testing
e. Exercises and review after 3 months

An alcoholic, very agitated, rushed to the emergency department with many complaints,
claiming if he is not attended to immediately he will jump in front of a car. What will you check
first?
a) Alcohol level
b) magnesium level ***
c) serum electrolytes (Na+ and K+)

Child at party drinks adult drinks. Now at emergency department confused, vomited, vitals
stable. What will you do find the cause?
Drug screen
CT scan of head

Child at party drinked alot of (adult drinks) exact words


Presenting with a long list of CNS and GIT symptoms with almost shock
What is the cause?
Alcohol(more chance)
Drugs
Aspiration of alcohol
Vomiting effect of alcoho
Old lady taking pcm 1000 pcm/codeine,Some other medicine for Pain in joints for long
time.Recieved pca morphine from friend who was on palliative rx for Cancer,now friend died
comes to you,for asking Morphine..whats the immediate management --
A refuse
B report to the drug prescribing authority about her drug seeking behaviour
C refer to rheumatologist
D increase dose of pcm / codeine
E Prescribe morphine

20 people attended a party. They eat 5 item & develop some gastrointestinal symptoms.
There is servay-
Food item with symptoms without symptoms total people eat
Bbq 10 7 17
Egg roll 10 2 12
Cheese 10 6 15
But last 2 are less relevent to me .which item cause this.
1.bbq chicken
2.egg roll
3. Cheese
. There has been an increase in the rate of alcohol related crimes in your town and you have
been asked by the local police to suggest means by which this can be reduced. What will you
do-
Make an advert about availability of an Alcohol Anonymous group in town
Advice police to gear up and punish offenders
Tell them doctors are not allowed to interfere with public issues
Organise a public talk about the health implication of alcohol abuse

woman with history of crohn’s disease on maintenance of prednisolone and mesalazine,now


presented complained of abdominal cramp and doctor decided to add azathioprine , what most
important test to do before this?
A-thiopurine methyl transferease-t's lethal if the patient has TPMT enzyme deficiency if receive
azathioprine
B-quantiferon gold-for TB as an alternative for skin test-in ppl recently taken BCG vaccine or ca
meds
C-LFT

Relation between Rota Virus & Low Birth weight?


Case Control
Cohort-depends on how qs is asked
Cross sectional
RCT

You are planning on conducting a research to look at the association between hypertension and
myocardial infacrtion. What is the most appropriate study design for this research?
a. Randomised controlled trial
b. Case study
c. Cohort***
d. Cross sectional
e. case control
t can be both, if hypertensive pt is selected to see dev of MI then its Cohort,but if pt's of MI are
studied to identify the cause as HTN then its case control.ques will be well described in exam

You want to make a study about asthmatic patients and if there have been any exposure to
smoke or its severity to exposure to smoke. What study is the most appropriate?
a. Case control
b. Cross sectional
c. Cohort ***
d. Observational study
Case to risk factor is case control and risk factor to case is Cohort study …
. Post operative patient became agitated after 24 hours.He tries to remove his Iv line.Cause?
Alcohol withdrawal- The most common cause of agitation for admitted pt is delirium which
come postoperative and alcohol withdrawal. So A is answer
Drug reaction
Psychosis
Dementia
Dx as Optic neuritis..Chance of MS in Next 10 years??
a.20%
b.40%
c.50%
d.70%
e.80%

MS scenario asking about progression of disease next 10 years.


a) 20%
b) 30%
c) 70%
d) 80%
e) 50%
. Patient with cellulitis leg , treated with cefazolin , and got relieved but 3 days later fever
develop with rash , In the future , which drug will be safe ?
A . Cefazolin -1st line cephalosporin
B . No cephalosporin
C , neither penicillin nor cephalosporin
D , Any cephalosporin except cefazolin

Hereditary spherocytosis present in grandfather and mum , what will be in patient


A.50%
B.100%
C.0%
Celecoxib, Perindopril, Indipamide presents with proximal weakness
celecox+ peri= hyper k= muscle weakness
celecox+indipa= hypo na= weakness-but also confusion etc

hereditary spherocytosis pt , now come with aneamia , low platelet count , but wbc count
normal splenomegaly 2 cm below costal margin , what is the cause of anemia ?
Hemolysis crisis
Aplastic crisis
Chronic occult blood loss

Hereditary Spherocytosis Child with URTI. Pallor no jaundice, Retics hight 6% , low Hb , Low
Platelet. On exam child had Pansystolic murmur at left sternal border. Cause
A Parvovirus
B. Hep A
C. ITP
D. Subacute infective endocarditis
E. Folate defiency
Very obese Self employed driver man come with his wife saying he snore a lot at night. Sleepy
often during driving . his wife say him to stop driving but he insisted on driving ( emphatically
reject ) , Next appropriate Mx
A. Ask him to stop driving
B. Inform driving license authority
C. Refer to dietitian
A 64year old man who experiences numbness and tingling sensations in his fingers especially
during winter. He experience occasional fever and has significant hx of smoking. Recently he
noticed that the skin of his hand is changing colour. What is the most likely diagnosis(see image
below)
a. Buerger's disease
b. Scleroderma***JM-307 ( Dx - ANA test ist 90%+Ve,RA factor+Ve in 30% and anticentromere
antibodies ( specific in 90% with limited Disease and 5% with diffuse
c. Hansens disease( Leprosy)

Q36)Elderly lady with X ray of hand.Case of rheumatoid arthritis.Most diagnostic test.


RF
Anti CCP***JM-368
ANCA
ANA
http://www.hopkinsvasculitis.org/types-vasculitis/rheumatoid-vasculitis
photo of hands with some post discolouration ( not a black sle fingers) but they said in recall he
has problem with finders during every winter time and his ANA is positive-
sle,
scleroderma***-bcoz raynauds more common in scleroderma
RA
A scenario of patient that was recently discharged from hospital from CCU for fainting attacks
and they added to him Amiodarone. He’s DM, HTN, IHD, Af on many medications ACEi,
Glibenclamide, Nitrates, Warfarin.
Then he developed acute painful thigh swelling, what to check for him?
A. INR*** pt on warfarin and amiodarone-hematoma
B. US
C. MRI
A pt. came for eye vision check for driving licence, and he wasn’t fit. He already went before to
another doctor and was unfit, but was given a correcting glass and the other doctor issue a fit
certificate for him. When you examined him again with the correcting glasses was still unfit. The
pt. decided to go to the driving licence authority with the other doctor certificate.
What to do?
A. Talk with the other doctor
B. Tell him he could drive but someone should be beside him
C. Report to driving licence authority that he’s unfit***

Old lady 80 years old presented by late stage Ca cervix, you told her everything about her
condition, and she accepted not to do surgery what to do?
A. Perform competence test
B. Arrange meeting with her family to decide
C. Ask for legal action ?? to decide who’s legally could decide for her (something like that)
D. Refer her to palliative care***

A lady was diagnosed with Ca, and her husband ask about her diagnosis. What to do?
A. Ask wife permission to inform her husband about her diagnosis***
B. Tell him
C. Ask for family meeting

. long scenario about multiple drug including : ramipril amlodipine , hydrochlorothiazide,


celecoxib, asking about which combination making muscle weakness without tenderness ?
A. ramipril and HCT
B. amiloride and ramipril
C. celecoxib and HCT
D.Amiloride and HCT***amiloride is k sparing diuretic thus amount of k in body is not removed
by kidneys and it is increased in body.

A scenario of a patient admitted for DVT and was started on Heparin and Warfarin. Then he
developed bleeding per rectum, BP 80/50, HB: 8.5 (was 15 at admission), and INR became 9.
What to do?
A. Give Vitamin K
B. Give FFP alone***-if alone is not mentioned then can go with this
C. FFP + platelets
D. Fresh whole bloood
( if in option FFP+vita K > choose it)
INR = 1-2 normal healthy patients but when patient are taking warfarin then it changes to 2-3.

INR above 8 without bleeding or with only a minor bleed (eg haematuria or epistaxis) - stop
warfarin, administer vitamin K1,
1-If the patient has life-threatening bleeding (eg intracranial or gastrointestinal haemorrhage):
Hospital management
cease warfarin
vitamin K1 IV
ptx
Fresh frozen plasma if no ptx.......VVVVVVVV IMP

https://transfusion.com.au/disease_therapeutics/warfarin/mgt-elevated-inr

A diabetic foot ulcer, diabetes controlled, dorsalis pedis and peripheral pulsation was felt, and
culture was taken. Picture nearly the same but less severe and less oedematous). What to do?
A. X ray foot***
B. Doppler US( done aftr antibiotic course if ulcer is not healing, if more than 6 wks then MRI)
C. Angiography
D. WBCs count

pt with fatigue, weight loss, fe decreased, ferritin increased, dx?


Anemia of chronic disease
myelodysplasia
Anemia of chronic disease……microcytic hypochromic anemia
Causes …any chronic disease e.g: Ra
Specific test….increased ferritin
A case of Diverticulitis with no previous history of DVT. Asking DVT prophylaxis.
Give enoxaparin until discharge**(enoxaparin should be given untill discharge if there is no
previous history of DVT. If there is previous history then it should be given for 10 days)
Give enoxaparin till 14 days
Give enoxaparin till 28 days
Fondaparinux till 28 days

Post-operative patient after LBO surgery with the past medical history of DVT. Other than
normal mechanical compression for prevention of DVT. What other treatment would prevent
post op DVT?
A. No treatment is necessary
B. Enoxaparin while bed rest
C. Enoxaparin for 6 weeks
D. Enoxaparin for 10 days****

Q83) patient with M.myeloma now present with constipation and weakness with
hypercalcemia.what is the next management ?
Ans- I/v fluid ****
B- hemodialysis (if creatinine level was raised we would have gone for hemodialysis)
pharma men to u for new drug,what u will ask from him:
a: which country
B. which journal published
c. when conducted
d.both group recived equal dose****
E. WAS THERE ANY CROSSOVER BETWEEN GROUPS

Cystic fibrosis .. One parent is carrier and the risk in child


(A) O***-Autosomal recessive patter,disease can come only if BOTH parents are CARRIERS
(b) 0.25
(C)0.5
Woman after admission to hospital become agitated within 24 hr. no hx of alcohol. Next
a. Blood alcohol check
b. Give o2*****
c. Ctpa
d.Vq scan

Emergency embolectomy indication -


Bruit( is the sign of stroke)
Paralysis***
6P's of arterial disease ..Pain,Pulseless,pallor,paralysis,paresthesia,Perishing cold ....indicates
severe arterial disease
JM 66 chapter page no.750( lower limb Ischemia)

a lady taking multiple drugs like for UTI and also another one infection….drugs r
gentamicin,amoxicillin,aminoglycoside?? .renal function is normal and she is well and all lab
investigation is normal except for K+ is 6.3 .what the cause
A delayed serum separation**** After blood drawing it took time to be examined and by the
time RBC broke and intracellular k+ released making it wrong interpretation as hyperkaelemia
B amoxicillin
C gentamicin
D Aminoglycoside
amoxicillin, TMP-SMX, ciprofloxacin, norfloxacin, nitrofurantoin together can cause
hyperkalemia. Amoxcillin alone can not cause hyperkalemia.
a junior doctor in foreign country did crime history..choices are
A inform immigration of the country
B inform Australia practitioners regulations****
C inform his medical defence agency
D inform hospital medical board
E take document for future

) a old man with diagnosed huntington few years ago and drive intentionally under alcohol
influence and get caught by police and presented to emergency department .he is agitated
when he is told for admission and want to go back home because lambing(yes lambing) season
is beginning.whats next
A inpatient detoxification at emergency department -bcoz huntington=dementia=involuntary
admission?
B outpatient detoxification and involved in alcohol counseling group***
C detained under medical mental
D refer to huntington support group
JM 162

man his father is dementia or Alzheimer ..he cannot take care of him anymore
A social work review*****
B immediately arrange a nurse to home care
C immediate arrange admission
D prescribe drug

old woman with 3 months ago had angioplasty of femoral artery and she had long standing
history of goiter .now she complaint of 5kg wt loss with normal appetite and increased
tiredness
Lab inx 3 months ago was including tsh glucose LFT TG cholesterol (all r non fasting blood
chemistry).All normal except TSH is low (like 0.01 or 0.1)...glucose was 7.2
So what investigation now is most appropriate for next dx for presenting condition?
A tsh****
B OGTT
C LFT

Q116) 12 week pregnant lady is coming for goiter …besides tsh what next important inx?
A t3
B t4***
C thyroid peroxidase antibody
D FNAC

a 5th generation Australian never travel abroad(it's said Australian not aboriginal ,sure) 3 weeks
chronic cough,non productive ,afebrile.Don't want his 3 month old son get infected.what to
inx?
A cxr
B pertussis serology***
C H.influenzae serology
D mycoplasma serology
E interferon gamma releasing assay for TB

)a man with BMI 36 and BP around 140 /90 and TG LDL cholesterol all raised ,liver function all
normal except ALT(not sure ALT or AST but sure is not lipase )200….what is the dx.
A pancreatic insufficiency
B chronic liver disease?
C metabolic syndrome**?
D cushing
E hypothyroid

Metabolic syndrome+high ALT = fatty liver

patient of DM came with glucose 2 mmol/l. what would u do?


JM 1411
a. IM glucagon**** or sc
b. IV dextrose 5 % bolus
c. IV dextrose infusion

pt with red raised nodules on leg.(sarcoidosis)


Acei inhibitor raised. most app Ix asked?
a. skin biopsy****
some others in option i don't rem. but not CT chest
Serum ACE levels are elevated in 60% of patients at the time of diagnosis. Serum ACE levels may
correlate with total body granuloma load.
Routine chest CT scanning adds little to diagnosis. High-resolution CT (HRCT) scanning of the
chest may identify active alveolitis or fibrosis and correlates with yield of biopsy.
Pulmonary function tests and a carbon monoxide diffusion capacity test of the lungs for carbon
monoxide (DLCO) may be performed-decreased
56 yr old lady with the following given ….BP around 130/95 BMI 24 waist circumference 73 or
83 cm glucose normal and serum tg (or cholesterol) raised.asking what would be the indicator
for an increase in her 5 yr cardiovascular risk(not cerebrovascular or health risk)
A HT > stroke
B blood sugar >dm
C serum TG (or cholesterol)> risk factor for the macrovascular complications of diabetes***
D waist circumference> cv risk A waist circumference of 102 centimetres (40 inches) or more in
men, or 88 centimetres (35 inches) or more in women
E BMI

Q185) Picture of Dupuytrens contracture and normal blood glucose next step
1. Physiotherapy
2. Remove flexor retina column
3. Steroid injection in tendon
Ans.3,jm 720

image of venous ulcer, a 64-year-old male with DVT presented with pyrexia 38.2 deg.
Management?
a. Dry dressing, Warfarin + mobilise wet dressing
b. Dry dressing, Antibiotic + mobilise
c. Wet dressing, Warfarin + immobilise
d. Excision of necrotic edges and then topical antibiotic
Right answer should be (wet dressing, warfarin, mobilize)**(JM 1352)
IF this option is not there then we will go for A.

atrial fibrillation and valvular dysfunction hypertension, surgery was done 3 days ago, now on
warfarin comes with 6cm pulsatile swelling in femoral region,vitals are moderate
next immediate management?
a. Compress the site-either blind or usg guided
B ultrasound***
C. ct scan
Pseudoaneurysm are common after catheterization procedures, Surgical repair of PSAs is rare
but is indicated in patients who exhibit rapid expansion, infection, compression syndrome, or
failure of Ultrasound-Guided Thrombin Injection which is first line of management

DKA initial treatment


Iv saline**( after that u will give insulin and after passing first urine, do the electrolytes so
according to k level infuse potassium chloride).

Hhorner syndrome question. Investigation asked=== CXR or ct


Here is an answer from Medscape:
A chest radiograph should be obtained; apical bronchogenic carcinoma is the most common
cause of Horner syndrome. If stroke is suspected, computed tomography (CT) of the head
should be performed.

lead poisoning you will find the following except ? level given in june ques 0.75
- Neurological manifestation
- Abdominal pain
- Diarrhea
- Hemoptysis****
If there is no hemoptysis in the options than choose liver enlargement
Adults:
Less than 10 micrograms per deciliter (µg/dL) or 0.48 micromoles per liter (µmol/L) of lead in
the blood
Children:
Less than 5 µg/dL or 0.24 µmol/L of lead in the blood

child with only APTT Increased and Factor 8 1%, no bleeding time given and the genetic
transmission is asked? Looks more like vwd than heamophillia so she chose
Autosomal dominant( if bleeding time is prolonged with normal platelet count then its more
likely to be VWD and if BT is normal with decreased platelet count then it would be
HEMOPHILIA A)
Go for x linked recessive-for haemophilia,can be differentiated from vwf only by vwf factor or

bleeding time.

Q215) 34. There was a question regarding p value for null hypothesis in a drug testing.
a. it will make no difference in p value if comparison of drugs
b. if p value is less than 0.05 then higher specific response***
c.if p value is <1.0 then means more difference in efficacy
d. if p value is 0 then both have equal response

A small p-value (typically ≤ 0.05) indicates strong evidence against the null hypothesis, so you
reject the null hypothesis. A large p-value (> 0.05) indicates weak evidence against the null
hypothesis, so you fail to reject the null hypothesis
P directly prop to null hypothesis but indirectly prop to drug efficacy
P α null h α 1/drug efficacy

Null hypothesis is called no difference or no effect of drug on the test


So if p value is less then reject null hypothesis meaning drug is efficient or effective orr has
effects.
If p value is more then accept null hypothesis meaning drug has no effect

in a case of scenario man described as having kleinfelter.. what would you do ?


Ans- Karyotype

Q232) following a perforated duodenal ulcer that got operated on and on is omeprazole and
rapid urease test done and it's negative . What follow up management do u do ?
Endoscopy in 6 weeks
Urea breath test in 6 weeks ***
Q233) case of a patient in warfarin and enoxaparin and INR 1.9( normal in s 1.2) what do u do ?
Cease enoxapArin and leave the warfarin***
Cease warfarin and decrease the dose of enoxaparin
Decrease warfarin
Decrease enoxaparin

5yr old child with go URTI 1week now pt is pale but no jaundice and spleen 2-3 cm palpable
a)aplastic crisis****( no jaundice,decreased rbc,hb,reticulocytes, initially normal or later
decreased platelet)
b)hemolytic crisis???( follows urti,jaundice, normal wbc,decreased platelet and rbc)

A 46 year old man came for URTI he has fasting glucose of 6.1 . OgTT done showed 6.7. what
next to do ?
a. repeat OGTT after 18 months
b. Fasting glucose yearly-if impaired levels
c. check fasting glucose 3 yearly**** jm 187- since his levels r normal
advise HbA1c
e. ogtt yearly

)diabetic ulcer picture was given. swabs taken. pulses present . what next ?
a. Xray***
b. MRI
c. Doppler
d. blood glucose levels

) 9 yrs old with type 1 diabetes. on small n intermediate acting insulin. every morning high
glucose levels. what to do ?
a. check blood glucose levels at 3 am****
b. check early morning insulin levels
c. increase the evening intermediate insulin
d. give insulin before breakfast
e. give another dose before sleep
The Somogyi effect is most likely to occur following an episode of untreated nighttime
hypoglycemia, resulting in high blood sugar levels in the morning. People who wake up with
high blood sugar may need to check their blood glucose levels in the middle of the night (for
example, around 3 AM). If their blood sugar level is falling or low at that time, they should
speak with their health-care team about increasing their food intake or lowering their insulin
dose in the evening. The only way to prevent the Somogyi effect is to avoid developing
hypoglycemia in the first place

A pharmaceutical company contact you about a drug which was developed and has been
approved by the council to distribute in the market. The drug reduces stroke risk in HTN &AF
patient with 40 % less GI bleeding. Which of the following is the most Important thing to
evaluate?
Common side effects
Absolute risk in the studied patient
Number of people involved in the trial
Cost of the drug

Hereditary Spherocytosis scenario. Which virus is responsible for it?


a. Parvovirus
Hs is hereditary where spherical rbcs are formed which lead to cell destructions and followed
by increased rbc production by spleen-splenomegaly.
c/f: The classic laboratory features of HS include the following [1, 2] :
Mild to moderate anemia
Reticulocytosis
Increased mean corpuscular hemoglobin concentration (MCHC)
Spherocytes on the peripheral blood smear
Hyperbilirubinemia
Abnormal results on the incubated osmotic fragility test
The major complications of HS are aplastic or megaloblastic crisis, hemolytic crisis, and
cholecystitis and cholelithiasis.
tx: splenectomy
In chronic congenital hemolytic anemia (ie, HS), long periods of asymptomatic disease depend
on a fragile equilibrium in which the excessive destruction of cells is balanced by accelerated
erythropoiesis. Disruption of this equilibrium can lead to rapid and dramatic falls in blood
hemoglobin levels, producing an aplastic crisis. Most, if not all, aplastic crises are caused by
infection with type B19 human parvovirus (HPV)

Hyponatraemia scenario given. Treatment was asked?


Chronic hyponatremia (> 72 hours’ duration) should be corrected slowly otherwise leads to
central pontine myelinolysis N:B:
Best fluid for hyponatremia......normal saline
Hyponatremia+confusion/ seizures/ any severe neurological symptom............hypertonic saline
(3%)
Rapid correction of hyponatremia...leads to..central pontine mylinolysis...... quadriplegia
Mild-anemia,nausea and vomitin,lethargy 130-134 mmol/L
Mod-personality change ,confusion,ataxia,muscle weakness 125-129 mmol/L
Severe-drowsiness <125 mmol/L
pt with haemophilia a prev no rx, not introduced to any factors(dnt get wat is saying-wether he
is nt introduced to any clotting factor or wat !!) he presented with knee pain with swelling
without any trauma.it happened for 2nd time.wat is most appropriate in management-
a. factor 7(vwb)
b.factor 8(hm 8)***
c.arthrocentesis

man want to go somewhere with increase hydatid disease prevalence.wats da advice


a.dnt bath in stream where animals bath
b.wash hand after in contact with dog***
c.take vaccine

coxiella burnetii- what causes infection-


a. factory worker
b.saw factory worker
c.dairy farmer****
Cattle, sheep, and goats are the primary reservoirs although a variety of species may be
infected. Transmission to humans occurs primarily through inhalation of aerosols from
contaminated soil or animal waste.
c/f: headache,pneumonia,hepatitis,carditis
Diagnosis: pcr blood and confirmatory IFA IGg
Rx: doxycycline and trimethoprim

old woman with low calcium.other blood report are normal.wats her cause of having low
calcium.cxray given-
a.dietary deficiency????
b.paraoesiphageal hernia
c. ca colon
d.chronic duodenal ulcer

Pt confused, high Ca 3.2 , creatinine 350, urea 18, K raised, Na 129, PO4 raised
a. Uremia***
b. met acidosis
c. hypercalcemia
d. hyponatremia
e. Hyperkalemia

Pt post laparotomy on PCA (pt control analgesia) program giving morphine 1gm.Now found out
pt is excessive drowsy but responsible to command. What next in mx?
A/ increase o2 supply via mask
B/Change morphin dose to 0.5 mgc.
C/Cease pca and give nurse supervised subcutaneous morphin on demand. *****
d.give naloxen

.a male with hereditary sperocytosis.how many of his sibling will be affected


a.25% of all male
b.25% of all siblings
c.50% of all male
d.none( 50% of all siblings)****

. A patient comes with tiredness for 6 months. Free thyroxine level is low, TSH is in normal
range. What investigation will you do?
A. Thyroid autoantibodies
B. Thyroid USG
MRI brain*****
D. Thyroxine 50mg
If T4 is low and TSH is low or normal, consider pituitary dysfunction (secondary
hypothyroidism) or sick euthyroid syndrome. jm 230.
Patient with claudication; DM (+), H/T (+), elevated cholesterol. What will you do to reduce
symptoms of claudication?
A. Lower cholesterol
B. Supervised walking****
C. Reduce his BP

Pt with claudication pain. Doc advises sx, but pt not fit to do surgery. He has DM, HT< obese.
What will you advice for long term management to reduce claudication symtoms?
supervise exercise
b. avoid smoking****
c. HT control
d. DM control

A man with inflammatory joint pain in multiple areas. Renal failure present. Which drug is CI?
A. Methotrexate***(contraindicated )
B. Hydroxychloroquine
C. Sulfasalazine
D. Azathioprine
E. Etanercept

A patient with chronic lung disease develops cough and fever. (?Acute on chronic bronchitis)
was admitted to hospital. Ipratropium and salbutamol given 8 hourly. Oral prednisolone
given.ABGA done showing: pH 7.35, PaO2 80mmHg, PaCO2 50mmHg, HCO3- 35mmHg .What is
your next management?
A. Give IV hydrocortisone
B. Increase bronchodilator to 4 hourly
C. CPAP***
D Admit to ICU
E. Change to iv cefotaxime

https://www2.health.vic.gov.au/hospitals-and-health-services/patient-care/perinatal-
reproductive/neonatal-ehandbook/pathology/blood-gas-interpretation

. A woman comes with the complaint of weakness, she is taking duloxetine, amitriptyline,
paracetamol & tramadol. What combination is causing this?
a. duloxetine & amitriptyline****(SSRI+MAOI,SSRI+TCA,SSRI+opiates)
b. amitriptyline & paracetamol.
C. amitriptyline & tramadol
d. duloxetine & paracetamol
http://www.emedicinehealth.com/understanding_antidepressant_medications/page9_em.htm
#bupropion_wellbutrin_wellbutrin_sr_wellbutrin_xl

) pt wz ACTIVE RA and chronic pyelonephritis , feels tired and lethargic , labs is done and it
shows " that's exactly as it was "
A- HB is around 9
B- MCH is low normal
C- Serum iron decreased
D- Serum ferritin normal
E- TIBC decreased
What is ur next appropriate management?
A- Packed RBCs
B- IV iron
C-low dose corticosteroid
D-Erythropoietin Stimulating Agent****

erythropoietin (EPO), a hormone that is produced by the kidney.


ESAs are usually given to patients who have chronic (long-lasting) kidney disease or end-stage
renal (kidney) disease. These patients usually have lower hemoglobin levels because they can't
produce enough erythropoietin.

An elderly 68years old man who is living alone, taking perindopril, B blocker diuretics for
CCF,Elderly care service visit him once 2 weeks for medical checkup. What is the greatest risk
factor for fall in this man?
a. health care service visit him not frequent enough
b. taking more than three drugs****
c. living alone
health care service being too far away from him

A man presents with breast enlargement on one side. He also has CHF, taking perindopril, b
blocker, digoxin for 2 years. Recently he developed dyspnea & oedema, so added
spironolactone for six months.He has family history of breast CA in his mother & a sister. What
is the cause of his presentation?
a. Digoxin
b. spironolactone****disco me( digoxin, isoniazid, spironolactone, cimetedine, oestrogen,
methyldopa)
c. breast CA-strong brca gene,unilateral enlargement
Spironolactone, an aldosterone antagonist used for high blood pressure treatment, presents
both anti-androgenic and oestrogenic activities. This drug can induce gynecomastia after one
month only of treatment, with a dose-dependent incidence.
But The mechanism of digoxin-induced gynecomastia is believed to be a direct action at
estrogen receptors due to similarity in the structure of digoxin and estrogen.
. A picture of a man with enlarged breast bilat, alcoholic has bilat swelling of parotid glands, Hx
of rheumatoid taking medication on, long Hx of gastroesophageal reflex taking ranitidine for,
heavy smoker, HT well controlled by medication. Asking whats the cause
a. ranitidine-also can cause
b. liver cirrhosis
c. anti HT medication
d. smoking

70 years old man presents with c/o right nipple swelling, no discharge . Underlying Htn,
diabetes, CCF, AF on Ramipril, bisoprolol, digoxin , statin for cholestrol. O/E : tenderness over
the breast. Asking diagnosis?
A. Digoxin toxicity***
B. Hypogonad
c. Androgen deficiency

) Picture of gynaecomastia. A 16 yrs old boy presents with this unilateral problem. He is
asthmatic and takes salbutamol and fluticasone for long time. What will be your advice?
A. Reassure as the problem will resolve***unilateral ??
B. mastectomy
C. Testicular hormone injection
D. Change the anti asthma drugs

Table 1.
Drugs that can cause gynecomastia
DRUG MECHANISM
Amiodarone Unknown
Calcium channel blockers (diltiazem, verapamil, nifedipine) Unknown
Central nervous system agents (amphetamines, diazepam, Unknown
methyldopa, phenytoin, reserpine, tricyclic antidepressants)
Cimetidine Androgen receptor antagonism
Cytotoxic agents (alkylating agents, vincristine, nitrosoureas, Primary hypogonadism due to
methotrexate) Leydig cell damage
Flutamide Androgen receptor antagonism
Hormones
• Androgens Aromatization to estrogens;
other mechanisms?
• Estrogens Direct stimulation of the breast
• Human chorionic gonadotropin Stimulation of testicular Leydig
DRUG MECHANISM
cell estrogen secretion
Isoniazid Isoniazid
Ketoconazole, metronidazole Inhibition of testosterone
synthesis
Marijuana Androgen receptor antagonism
D-penicillamine Unknown
Phenothiazines Elevated serum prolactin
Spironolactone
theophylline

2 A 17 y.o. football player presents with gynaecomastia. He is on inhaled steroids for


asthma.What is the cause of his gynaecomastia?
a. normal adolescence
b. steroids
c. anabolic Steroids***
)(anabic steroids) the question seems to imply he also uses anabolic steroids as a sports man
which may have been omitted here, otherwise idiopathic. In Handbook, it was just the picture
and the age with no other hx thus idiopathic.

years old woman, suffering from chronic rheumatoid arthritis, recently develop hypertension
came for the treatment? She describe past history of asthma and pylonephritic syndrome.
a.) Amilodipine****ccb
b.) Ramipril
c.) Lorsatan
d.) Metopropanol
Uptodate: The calcium channel blockers (especially those of the dihydropyridine group, such as
nifedipine and nicardipine) are excellent agents for the treatment of hypertension in asthma. In
addition to effectively lowering the blood pressure, they also have the theoretical advantages
of opposing muscle contraction in tracheobronchial smooth muscle, inhibiting mast cell
degranulation, and possibly reinforcing the bronchodilator effect of beta agonists.The use of a
low-dose thiazide alone or with a calcium channel blocker represents the preferred regimen for
the initial management of the hypertensive asthmatic.

A lady of 45yrs of age complaining of weakness, lathergy and constipation for 6months.She is
gaining weight despite trying to reduce weight.O/E her BP is 100/70mmhg and pulse was
55beats/min.what is the most probable diagnosis?
A. DM
B. HYPOTHYROIDISM***
C.Cushing’s syndrome
18 Amenorrhea with high TSH asking cause

Patient presented with amenorrhea for 12 months. BMI 29. lab are given FH 3.5(normal) , LH
normal . prolactin 620 (150 to 700) TSH 14 high. Cause asked
A.Hypothalamic dysfunction
B.Pituitary microadenoma
C.Primary hypothyroidism***
D.PCOS

laparotomy indication
After RTA .. which is indication for urgent laparotomy
1- subcapsular liver hematoma
2- splenic hematoma
3- retroperitoneal collection
4- retroperitoneal free air***

A man in your GP clinic was advised for CT abdomen.Then he asked about it’s radiation.How
many Xrays radiation are equivalent to this investigation in case of radiation?
A.50
B.100-ct head
C.400****

A Tongan boy after a violence with peers was brought to ER by his friends.O/E his vitals are
stable there is significant swelling in left eye and cheek region.He doesn’t understand
English.you know a hospital security can speak tongan and also a nurse knows tongan very
well.your official interpreter was busy with another call.what will you do?
A. call the nurse to interpret??
B.call the security guard to interpret
C. wait for official interpreter to finish his current call and then next for your patient***
D.ask one of his friends to interpret

GBS case with limb paralysis and mild dyspnea. What is the most important next step :
A. LP-to diagnose
B. Spirometery***-to assess resp function
C. MRI

Routine test for GBS has done at 36 weeks pt found to be GBS carrier
Prophylaxis of which antibiotic will be done if pencillin hypersensitivity is present
Cephalexin
Amoxicillin
Clindamycin****
Vancomycin
https://www.cdc.gov/groupbstrep/guidelines/downloads/recommended-regimens.pdf
For penicillinallergic patients who do not have a history of those reactions, cefazolin is the
preferred agent because pharmacologic data suggest it achieves effective intraamniotic
concentrations
First check if anaphylaxis to cephalosporin too if no then give cephazolin/cephalexin/any
cephalosporin if yes then check for clindamycin.

. A patient presented with macroscopic hematuria. 6 months back he diagnosed as c-ANCA


positive vasculitis causing hematuria and started on Prednisolone and Cysclophosphamide. The
RBCs on examination are 20% dysmorphic and 80% normal, what is the most appropriate next
step?
a) Stop cyclophosphamide***causes hemorrhagic cystitis
b) Repeat renal biopsy
c) CT abdomen
d) Renal USG
e) Cystoscopy

Recall of a male with burns in who overheard the staff talking about him and thus requested to
be discharged home
a. Allow him home and see him at OPD for follow up
b. Refuse
c. Refer case to management***

A case with genital ulcer with h/o multipartner and signs of meningitis.on CSF raised
glucose,polymorph nuclear cells predomonantly.which organism responsible for?
1.HSV
2.HIV????
3.pneumococcus
4.meningococcus
5.treponema pallidum????-syphilis

5 yr old boy repeated abscess infections scenario.wat will u find in investigation


a.neutrophil count ***
b antibody deficiency
c.complement level

Patient on medications for HTN. She feels dizzy when get out of the bed... happening for a
while...She also had some falls. BP 160/100 JM 530.Has AF. Next Ix
A echo
B xray
C holter monitoring
D 24 hour BP monitoring
E check BP again in supine position

patient history of surgery for perforated diverticulum disease , post op 2 day , get confused and
oxygen 88% chest examination normal. Most appropriate next invx
a chest x ray ***-atelectasis rx:high flow o2 and cpap
b CTPA (2nd)-for embolism
c blood glucose
d Blood alcohol level

18 years old boy back from camping trip c/o diarrhoea and vomiting. After 24 hours presented
with chills, rigor, fever, traces of blood in stool. Given only systolic pressure of 50mmHg,
abdominal pain, . All the the investigation were normal except platelet 20, Hb was 9 . Asking for
diagnosis:
A. Hemolytic uremic syndrome****-diarrhea due to infection with e coli
B. Disseminated intravascular coagulation-in injuries,obs causes, brain injury or cancer. Low
plt,low fibrinogen, no clotting, low d dimer.
C. ITP-presence of purpuras ,no diarrhea etc

pt on sulphasalazine , now got PAINFUL ulcers , plus gingivitis , platelets normal , rbc normal ,
wbc may be dec i forget , but was dec , other ev thing was normal cause a
A,sulphasalazine toxicity ®**** (coz of neutropenia).
b herpes simplex ulcer
c acute leukemia

) A hypertensive and diabetic presented with histories of numbness, nausea and sweating. He
developed dizziness while urinating. Pulse is irregular and ECG shows atrial fibrillation. Other
things said. Diagnosis asked
a. Myocardial infarction*******(Pulse is irregular A.F causes M.I).
b. Micturition syncope
c. Vasovagal syncope

Overdose of Amitriptyline with widened QRS complex. Patient is drowsy and hypotensive. What
is your next management?
a. Oral activated charcoal-early stages if no seizures and cva toxicity
b. Dialysis
c. Gastric lavage
d. Intubate and give sodium bicarbonate***- Serum alkalinization with intravenous sodium
bicarbonate has been the mainstay of therapy in TCA-induced cardiovascular toxicity.
Prolonged QRS is most often the indication for serum alkalinization in TCA toxicity.
e. Cardioversion

Q) Picture of necrosis on 3 fingers, patient with rheumatoid arthritis, non smoker, can't
remember the rest of the description, but asks for diagnosis
A ana*****(secondary Raynaud's). –secondary to rheumatoid vasculitis-tested by ANA &ANCA
B ena-SLE
C anti ssa-for SLE,SSB-sjogrens
D anti citrulline-for rheumatoid arthritis-early

A patient who has rheumatoid arthritis had history of DVT. She has been a regular patient with
clots forming at many places in her body. Now he came with black fingers . what is the
investigation to find the underlying cause?

A- ANCA-for r vasculitis
B- Anticardiolipin-for antiphodpholipid syndrome
C- ANA-also for r vasculitis
D- Anti ds DNA-for sle

https://www.hopkinsvasculitis.org/types-vasculitis/rheumatoid-vasculitis/

Woman from polynesian island early 30s , Cook or something recall ( prone to DM) asking for
screening
a) OgTT 2 yearly
b) RBS yearly
c) RBS 2 yearly****-every 3 yearly rbs or glycated Hb
d) Fasting yearly-since cook islander
e) Fasting 2 yearly

A 35 year old woman presented for screening because of a history of diabetes in her mother.
She is otherwise fine. Her BMI is normal. All investigations were normal. What will you do?
JM 187
a. RBS every 2 years
b. OGTT every 2 years
c. Observe till age 45, then 3 yearly blood sugar
d. FBS now
e. FBS every 3 years****(every year in aboriginal and very high risk)

5. Man with epilepsy one one drug has seizures, changes to Carbamaz, asking when fit to drive
(seizure free period)
a) 12m
b) 6m
c) 2 years***
1 year for chronic seizures
3 months free for new pt,
for applying for LL 2 years seizure free
https://www.epilepsyact.org.au/wp-content/uploads/2014/06/Driving-cut-down.pdf

New blood test for Carcinoma of colon. Graph given.


Asking the point of 100% Specificity.
https://goo.gl/images/r4ocLm
B= 100% sensitivity. D= 100% specificity.

. The cut off point of screening test is increased.


both sentivity and specifity will increased
B Sen increased, Sp decreased
C.Sen decreased, SP increased

one population , mean systolic BP is 115 and SD is 15 so what will be range of 95% population’s
BP?
a) 100 to 130
b) 85 to 145-68% within 1 SD, 95% within 2 SD & 99.7% within 3 SD, 95 percent are within two
standard deviations (μ ± 2σ),Mean=115+2SD =115+30=145& (μ ± 2σ),=115-2SD=115-30=85
C)105-140
33. The incidence of ca carcinoma is increasing in community but prevalence is decreasing what
does it mean?
a. screening test is not efficient in diagnosing ca lung at early stage
b. morbidity of ca lung is increasing
c effective treatment of ca lung is available
Prevalence of cancer refers to the number of people alive with a prior diagnosis of cancer at a
given time.
if morbidity (not mortality) increases prevalence will rather increase but not decrease

Pt with active RA on methotrexate ,feels tired and lethargic labs done showed the ff
Hb 9.5 g/dl
Mch low normal
Serum iron decreased
S.ferritin normal
Tibc decreased
What is treatment of choice
A. packed RBCs
B. iv iron
C. low dose corticosteroid
D. erythropoietin injection.****
E. Folic acid
9yr old girl presents with pallor patient with picture of microcytic anemia MCV low Serum
ferritin low. TIBC high. Dx
A) iron deficiency anaemia
b. Megaloblastic anaemia
C) sickle cell dx
D. Lead poisoning
E. Thalassemia

. A man presented for a sick certificate but presented with ataxia with alcoholic fetor asking
most important thing to address
a. alcohol counselling programme.
b. reassurance
c. CAGE PROTOCOL*****

Q69. A man presented for a sick certificate but presented with ataxia with alcoholic fetor asking
most important investigation to do
a. ct scan of brain
b. LFT**** -EtG most highest sensitivity and specificity,then CDT ans GGT for liver damage.
c. INR
d. fasting blood sugar
e. full blood exam

A case of recurrent herpes zoster infection. Tzanck prep(for herpes simplex and for chickenpox)
done showed multi nucleated giant cells. which treatment is best for resistant viral infections
A. High dose acyclovir
B. foscarnet -or cidofovir for acyclovir reisitant HS
C. valaciclovir****
D. Ganaciclovir

. Microcytic hypochromic anemia at child asking for investigation


a) Serum Ferritin
b) HB electrophoressis***
woman presents with her husband with sleep disturbance. He snores and is overtly obese.
What is the best next step in management?
a. Sleep hygiene****
b. Uvulopalatoplasty
c. Refer for aggressive weight loss scheme

Lady presented with headache affecting her in school. She wants an excuse duty because she is
lacking good sleep which is affecting her. She does not want medications. How will you help
her?
a. Advice sleep hygiene***

A man was brought to the ED after being involved in an MVA. He initially lost consciousness but
regained it and was well until he started having seizures. Some labs given. What is the cause of
the presentation?
a. Epilepsy
b. Intracranial hemorrhage
c. Inappropriate secretion of Antidiuretic hormone***(SIADH causes hyponatremia and
hypernatremia causes epilepsy).if hyponatremia in labs

A scenario of low sodium level – SIADH. Cause asked


a. Frusemide
b. Thiazide****

. A Man on vancomycin. He starts to develop a rash following the infusion of Vancomycin. On


stopping the drug the rash starts to go down. Which of the following is the appropriate action?
a. Stop vancomycin-first stop the infusion IMMEDIATELY then give IV antihistamines then
resume infusion slowly.
b. Slow the infusion rate for vancomycin
c. Give prednisolone

old woman presented with c/o lathergy and fatigue. CBC shows hypochromic and microcytic
anemia no h/o abdominal pain or altered bowel habits or bleeding per rectum. What you will
advice next?
A) Serum ferritin
b)fecal occult blood
c) hb electrophoresis
d)colonoscopy-suspecting ca colon

a pt known case of DM she had an episode of Afib in the past due to which she is on warfarin.
Her other medications were perindopril amlodipine indapamide. He now started on gliclazide.
She came to u with c/o hypoglycemic attacks 2 episodes whch one of following is the culprit
drug?
A. Amlodipine
B metoprolol
C indapamide
D perindopril
E warfarin-warfarin with sulphonylurea causes hypoglycemia
(She was not taking metformin not mention of metformin in scenario)

A patient who is hypertensive hyperlipidemic well controlled with Atoravastatin, Amlodipine,


Perindropril, Aspirin and indapamide. He was recently diagnosed with DM and was prescribed
Glicazide. He developed 2 hypoglycemic attacks. What is the reason for hypoglycemia?
A Perindropril
B Indapamide
C Aspirin
D Atorvastatin
E Amlodipine

a pt has facial nerve palsy you nd one of ur enterny has taken video and after pts permission
use it foa a conference. What is best regarding confidentiality of pt?
a) discuss it with ur collegues and show them
B) download to pts medical record and delete from mobile
c) use it for every conference without more permission of pt
d)can discuss to medical board without permission

Lady come complaining of increase in size of her limbs, u examine her: bilateral lower limb
edema non pitting from foot to groin what investigation?
Urine protien
Ct abdomen pelvis-lymphedema due to some ca
Echo heart
Liver function
Ultrasound Abdomen

Patient with bilateral non pitting edema up to groins.inv??


A.duplex both legs
B.ct scan abd & pelvis
C.echo
Man woke up at 4 am due to tongue swelling. He had chronic hay fever n shell fish allergy. He
went to asian restaurant the previous evening. What is the cause of his tongue swelling?
A accidental shell fish ingetion
B c1 esterase deficiency
C idiopathic anaphylaxis

. Patient with a fracture managed on Fentanyl infusion for pain. He complains of continuous
nausea but the pain is well controlled on Fentanlyl. Most appropriate next step :
a. Reduce dose of Fentanyl
b. Add IV Metoclopramide along with Fentanyl infusion
c. Replace Fentanyl with Morphine
d. Something metioned about giving Tramadol
e. Continue same dose of Fentanyl

A woman had Tb I guess but the child got TB and was strongly positive for quantiferon test .
What to do next most app and investigation was found isoniazid sensitive . No option Of admit
and obeserve ,
a. do Mountox test ,
b.start isoniazid course . . .

A previously fit 55-year-old man has undergone an emergency right hemicolectomy for a
perforated caecal carcinoma. Two days after the operation you note the following on his fluid
balance sheet – intravenous input 2 L, nasogastric aspirate 2 L, drain losses 700 mL, urine output
500 mL. Biochemistry shows [Na+] 135 mmol/L, [K+] 3.0 mmol/L, [Cl−] 100 mmol/L, [HCO− 3 ] 27
mmol/L. Which of the fluid balance regimens below would you order for the next 24-hour
period?

a. 2 L N saline + 3 L dextrose 5% + 50 meq KCl


b. 2 L N saline + 1 L dextrose 5% + 50 meq KCl
c. 1 L N saline + 3 L dextrose 5% + 100 meq KCl
d. 3 L N saline + 2 L dextrose 5% + 100 meq KCl
e. 1 L N saline + 1 L dextrose 5% + 100 meq KCl

A man came for sick certificate already taken many leaves in the past 3 months something , takes
11 standard units of alcohol on weekends . diagnosis ?
A. alcohol dependence.
B. Alcohol abuse

And in another scenario of patient with DM , hba1c was 11 and FBs also raised asked cause .
a. Metformin toxicity,
b. insulin resistance ,insulin insuffficiency
c. insulin toxicity ,
d. renal failure

Man returned from Thailand with fever, three episodes of loose stools . How to exclude malaria
from the given options ?
1.diarrhoea ,
2.fever ,
3.without splenomegaly
A man went to asia had fever in 3rd day with chills vomiting .. high fever continued all days ..no
splenomegaly ... abdominal pain .. diarrhea
Hb decreased
Mcv 87
Wbcs normal
Platelet decresed

What in senario will help you to exclude malaria


1 incubation period short only 3days
2 daily fever
3 abscence of spleenomegaly
4 low platelets
5 irrelevant
The time from the initial malaria infection until symptoms appear (incubation period) typically
ranges from: 9 to 14 days for Plasmodium (P.) falciparum. 12 to 18 daysfor P. vivax and P.
ovale. 18 to 40 days for P. malariae

DM 1 pt after a penetrating injury developed swelling of ring finger, it was tender esp on flexor
side pt had problem with extension of the finger ... What to do next
A. IV cephalothin(exact wording)
B. Iv flucoxacillin
C. Surgical exploration
D. Iv cephalothin with physiotherapy
E. Iv flucoxacillin with physiotherapy

12-old age woman after git operation 12 hr became agitated and disoriented in place time but
not person what to check is imp for her condition a.serum elect. B.bld culture. C.urine
culture(the ans for delirium in old age)

new question scenario of woman with Alzheimer's fall and has fr neck femur after the
admission she develop signs of delirium her drug hx include
metformin..alprazolam...risperidone...dethiopine...donopezil asking what to do
A.replace alprazolam with diazepam
B.cease alprazolam
C.cease risperidone
D.cease dethiopine
E.cease donopezil

Pt with 2 ulcers, one on leg above medial melleolus and other on plantar surface of foot over
the head of 2nd metatarsal. Burger test positive of that limb. H/O claudication while walking,
and rest pain as well. And pulses are not palpable for this limb. Asked reason behind the pain?
a) Ulcer
b) positive Burger test
c) Osteomyelitis

muscle weakness scenario asking for investigation: muscle biopsy ??


106. old patient on a number of drugs including ACEI, ASPIRIN, METFORMIN, statin, presenting
with muscle weakness. Which drug is responsible?
A)Statin
B) ACEI
C) Metformin
D) Aspirin

. Image of bilateral gynecomastia, more on the left side.


Male, 54 years, heavy drinker, 2 to 3 liters per day. He is having diffuse abdominal distension
and bilateral parotid swelling and hand tremor. He is also having small Testicular swelling.
Diagnosis?
a.Malignant teratoma
b.Hepatic liver disease
c. Leydig cell tumour
d. Colon cancer

there was 1 question related to flu like symptoms body ache, high fever, rash, vomiting
diarrhea Bp 80/50 diagnosis?
a.malaria b dengue c hepatitis d. toxic shock syndrome(coz of low bp i chose this)

. pt presented with fever 5 days after coming back from Cambodia Jaundice with elevated liver
enzymes,low platelets, no rash or arthralgia, wt is the diagnosid
A. Malaria –in south east asia+, low plt
B. Dengue fever
C. Hep. A

Most common cause of pt present with DKA?A)previously undiagnosed IDDM.B)omit insulin.

14yrs old boy,One of your long term patient since birth,presents to you at your surgery after
Dog bite at home,when he was playing with it , accidentally fell over the animal and provoked
it. O/E There were Two deep puncture wounds. No deep structural injury.His last immunisation
for tetanus was at 4 yrs of age according to his File.You advise him to get the wounds cleaned
and apply non-Adherent , absorbent plaster by the Office nurse and give him prophylactic
antibiotics for 5 days. As the bytes are deep puncture wounds you decided to give him tetanus
injections as well.
Which one is the BEST vaccination?.
1.DTPa(Diptheria,Tetanus,Pertussis acellular)
2.Tetanus Toxiod-Tetanus toxoid should be administered if 5 years since the last dose and the
patient has completed a full primary course of tetanus immunisation
3.Adult Tetanus and Diptheria Toxoid - >10 yrs
4.Tetanus immunoglobulin + Tetanus toxoid-in unvaccinated
5.Diptheria toxoid +whole cell pertussis +Tetanus toxoid (DPT) booster.
https://www.racgp.org.au/download/Documents/AFP/2009/November/200911dendle.pdf
dTpa-booster given at 10-15 years-correct answer in this case.if not given then chose dT

A 50-year-old male is brought by ambulance after a motor vehicle accident.On assessment he is


found to have compound fracture of the shaft of the femur.
He received his three“tetanus needles “as a child.He is unable to recall the time when he had
his last tetanus booster.
Which of the following is true regarding tetanus prophylaxis in this patient?
a. No vaccination is required
b. Give tetanus-diphtheria (Td) booster
c. Give tetanus toxoid
d. Give diphtheria-tetanus-pertussis (DTP) booster
e. Give tetanus toxoid and tetanus immunoglobulin

http://www.immunise.health.gov.au/internet/immunise/publishing.nsf/Content/Handbook10-
home~handbook10part4~handbook10-4-19#4-19-9

. Patient from Iraq.Gives history of dry cough.No fever not hemoptysis.You notice pan systolic
murmur at apex.Chest x ray given.Diagnosis?
Tb
Mycoplasma Pneumonia
Rheumatic fever
Ca.Bronchus

.new q— scenario of acromegaly,asking inv?


Ans.insulin like growth factor(I opted this one bcoz its key test for acromegaly),sorry forgot
other options
Sx: swelling of soft tissue in the hands and feet (onset signs), enlarged bones in the skull, face,
jaw, hands and feet, joint pains, pins and needles in the hands, headaches, gaps forming
between the teeth, which may cause a ‘bad bite’, barrel chest, enlarged heart (cardiomegaly),
thick and oily skin and strong body odour, growth of skin ‘tags’, overgrowth of hair, husky
voice,enlarged tongue and lips,snoring or drooling while asleep,heavy sweating
(hyperhidrosis),vision changes, such as loss of peripheral (side) vision
Inx: IGIF,GF,CT MRI for adenoma which is usually the cause
Rx: drugs(somatostatin analogues (trade names Sandostatin and Lanreotide),
surgery,radiotherapy

.multiple myeloma scenario with long lab value(urea & creatinine was really high),asking next
most appropriate mx??
a.hemodialysis
b.normal saline
c.electrolyte containing saline
ans.a (bcoz RF occurred already ,& RF is very common in MM)
indegenous man brought by wife has joint pain for last2 days,fever for 3 days,wife noticed a
skin rash on his back,he has a sore throat about 1 week ago,there is a long PR on ecg (typical
scenario about RF),,asking most appropriate inv??
a.blood culture
b.urine culture
c.rheumatoid factors
d.antistreptolysine antibody
ans.d(I opted)
***no FBC,ECHO,throat swab in options
. surgeon operating in emergency surgery (splenic rupture) ,needle stick injury, what to do
immediately
a. ask anaesthetist to draw blood from pt for analysis
b. ask anaesthetist to collect blood sample from surgeon
c. arrange for antiretroviral treatment
d. wash with povidone iodine solution
e. rub hand with alcohol swab, wear double glove

scenario of concussion, followed by polyuria,polydipsia


asking for diagnosis
a.SIADH b. diabetes inspidus

New ethics: A child with severe asthma was brought to emergency hospital by ambulance.
When measured the SP02 level very low, the patient was very dyspnoeic with gasping ,
comatose, silent chest. The oxygen cylinder in ambulance not working. Child went to apnea.The
parents were distressed and aggressive and asked for compensation.
1) The hospital manager will be responsible and face criminal case
2) No court case but hospital has to pay compensation
3) The doctor is responsible to investigate the case & report parent
4) It’s just an accident. So nobody responsible
5) The ambulance driver will have to attend civil court.(I choose this but not sure)

9)New Ethics: You are working in a busy emergency department and there were rush of
patients at once while still there were other serious patients in the emergency. Your
department head is on hospital meeting & you are one of the junior doctor at that period.
Which of the following patient will be your next priority.

1) Patient aggressive & abusive with multiple laceration & psychotic behaviour
2) Case of acute poisoning with frothing from mouth
3) Case of severe chest pain with syncope (I choose this but very confusing)
4) MVA with severe bleeding all from the head
5) Burn patient with 36% burn in surface area.

There was a mother who brought her child for examination frequently after birth. Child looks
normal but the mother drinks beer, smokes cigarette and occasion some drugs(Might be
forgot). There are bruises in mist part of her body. She sleeps in floor in her uncle’s house.
Regarding this presentation where should your immediate first call be directed to?
a) Drug & rehabilitation centre
b) Child protection centre
c)Hospital child social service
d) Emergency Housing (I am not sure, I choose this)
e) Police Station

A post marketing drug surveillance study of a new heart failure therapy to the market
was carried out on 10,000 subjects who had completed clinical trials. which one of the
following most accurately reflects the information genereated from such a study?
1. Adverse events profile
2. comparative therapeutics efficacy
3. cost benefits trial
4. cost effectiveness
5. Drug potency

A 28-year-old lady comes to the physician to explore the cuase of an endocrine disorder.
Physical examination reveals a solitary thyroid nodule. Laboratory studies showed an increased
serum calcitonin level and a gastrin-induced rise in the secretion of calcitonin. A biopsy
confirmed the presence of a carcinoma. The patient is scheduled for a total thyroidectomy.
Which of the following is a potential complication of this treatment?
(A) Acromegaly
(B) Cretinism
(C) Hypertension
(D) Hypoparathyroidism
(E) Renal osteodystrophy

known hyperparathyroidism developed perioral tingling after surgery with low Ca level given
a. Ca infusion
b. Ca carbonate
c. Ca + vit D
d. Oral Ca
if ca <1.9 then iv otherwise oral

What are the signs and symptoms of early sepsis?


Respiratory acidosis
Decreased cardiac output
Hypoglycemia
Increased arteriovenous oxygen difference
Cutaneous vasodilation

Das könnte Ihnen auch gefallen